Está en la página 1de 102

51.

A 30­year­old male presents to ER with complaints of recurrent seizures. Physical 
examination reveals candidiasis of oral cavity and cervical lymphadenopathy. On 
investigation, he is found to be HIV positive. An MRI of his brain with contrast is 
shown below.

 
What is the most likely diagnosis?
Answers
                A. Neurocysticercosis
                B. Tuberculoma
               C. Toxoplasmosis
                D. Pyogenic brain abscess
                E. Glioblastoma multiforme
Explanation
This contrast MRI shows multiple ring­enhancing lesions. The patient also has oral 
thrush and lymphadenopathy. This constellation of signs and symptoms points to 
HIV­associated toxoplasmosis as the most likely diagnosis. Seizures are a common 
complication of such brain lesions because of the surrounding edema that they 
produce. The diagnosis can be established by serological methods. Detection of IgG and
IgM antibodies to toxoplasma is diagnostic of an acute infection. Commonly employed 
methods for detection of antibodies is Sabin­Feldman reaction, indirect fluorescent 
antibody test and double sandwich IgM­ELISA (Choice C)
Choice A: Neurocysticercosis can also cause multiple brain lesions. The lesions are 
usually cystic. One or more calcified areas may also be seen that may be typicaaly 
cigar shaped. Sometimes cysticerci are visible on MRI. Neurocysticercosis is 
uncommon in the United States. It should be suspected in immigrants from developing
nations.
Choice B: Mycobacterial tuberculomas can also cause similar lesions but, the disease is
uncommon in HIV negative individuals. In HIV positive individuals it does come into 
differential diagnosis, for which a biopsy would be essential.
Choice D: Pyogenic abscess of brain is uncommon lesion in USA. More commonly brain
abscess is produced by Toxoplasma, Aspergillus,Nocardia and Mycobacteria. All would
come as a differential diagnosis in HIV positive patients. Pyogenic abscess often show 
daughter abscesses in relation to the main abscesss.
Choice E: Ring­enhancing. glioblastomas are usually solitary and have  a 
characteristic butterfly appearance
Educational Objective:                                  
The finding of multiple ring­enhancing lesions in an HIV patient is most likely due to 
toxoplasmosis. The diagnosis can be confirmed by serological tests for specific IgM and
IgG antibodies.
 
 
52
A 6­year­old boy ­ a recent immigrant from Pakistan­ complains of difficulty in 
walking. He had fever a few days ago that was accompanied by headache and 
lethargy. On physical examination, there is a definite weakness, more on left legs and 
is flaccid in nature. Reflexes are diminished in the affected region. Lumbar puncture 
shows lymphocytic pleocytosis, normal glucose and mildly raised proteins. PCR assays 
yield viral RNA from the CSF. This patient has an infection that was most likely 
transmitted through which of the following routes?
Answers                
                Α. Blood transfusion
                B. Respiratory
                C. Sexual
                D. Insect bite 
               E. Fecal ­oral
 
Explanation:
Fever, headache, photophobia and painful movements of extraocular muscles are 
classic symptoms of meningitis. His CF picture is indicative of aseptic meningitis. 
Isolation of viral RNA from the patient’s CSF indicates that an RNA virus was the 
most likely cause of his aseptic meningitis. Some viruses that produce aseptic 
meningitis are: enteroviruses, West Nile and other arboviruses, HIV, herpes simplex­ 
2, Epstein­Barr virus, Variclla­ zoster, mumps and adenoviruses. 
Enterovirus infection is the most common cause of aseptic meningitis accounting for 
up to 10,000 cases per year in USA. The enteroviruse are a family of single­stranded 
RNA viruses that include the coxsackie, echo and poliomyelitis virus. Enteroviruses 
are so named because of their fecal­oral route of transmission and ability to replicate 
in the GI tract. However, they do not produce gastroenteritis.
The patient described above appears to have poliomyelitis. Polio classically occurs in 
unvaccinated immigrant patients from endemic areas of the world. Symptoms of fever,
malaise and aseptic meningitis occur first, and can be followed by severe myalgias and
asymmetric paralysis that affects the legs more often. Damage to anterior horn cells 
produces flaccid paralysis. (Choice E)  
Choice A:  HIV and CMV are the viruses that can be transmitted by blood transfusions
and are capable of producing aseptic meningitis

Choice B: Varicella, mumps and adenoviruses are among the few viruses that are 
spread by respiratory ssecretions and capable of causing aseptic meningitis or 
encephalitis.
Choice C: Sexually transmitted viruses that may cause aseptic meningitis include 
HIV. HSV types 1 and 2, EBV and CMV. HIV and HSV­2 are amongst the more 
common viruses that produce aseptic meningitis. 
Choice D:  Arboviruses are transmitted by insect bites and can cause aseptic 
meningitis. These include: the togaviradae (Eastern, Western and Venezuelan equine 
encephalitis) and the burryaviridae (California encephalitis) families. These viruses 
are most common in the summer and fall when arthropods are most active.
Educational Objective:
Enteroviruses are the most common cause of viral aseptic meningitis. The enterovirus 
group includes the coxsackie, Echo and poliomyelitis virus. Polio virus can cause lower
motor neuron paralysis in addition to aseptic meningitis, especially in non­immunized 
individuals from endemic regions. Diagnosis is usually clinical. The virus can be 
cultured from the stool up to early convalescence. Four fold rise in complement fixing 
antibodies against type 1,2 and 3 polio virus at interval of three weeks can be used if 
desired.

 
53
Incidence of neonatal tetanus is on rise in a particular community. What would be the 
most appropriate measure to control this problem?
Answers  
                A. Routine vaccination at birth
                B. Post­ partum antibiotic administration to neonates
                C. Extended post­partum hospital stay.
                D. Early inception of post­partum breastfeeding
               E. Vaccination of young mothers and early start of breast feeding

Explanation:
In developing countries, most cases of tetanus occur in mothers with incompletely 
removed placentas and in newborns with unclean and infected umbilical cord stumps. 
Tetanus vaccine has been available since 1925. Immunity to cross infection is 
produced by vaccination with a formalin­inactivated toxin, also known as tetanus 
toxoid. The first vaccine dose is started approximately 2 months after birth. Vaccine is 
administered during childhood at 2, 4, 5, 15 to 18 months and at 10 to 12 years of age. 
Booster immunizations are required every ten years to maintain protective levels of 
antibodies. Long term immune response to tetanus is usually mediated by IgG 
antibodies. As IgG antibodies can cross the placenta, a newborn baby will have 
protection from maternally derived antibodies, if the mother has immunity against 
tetanus. Hence vaccination of young mothers along with inception of breast feeding 
from just after birth would be the most appropriate strategy for preventing neonatal 
tetanus. Immunization of the baby at birth would not suffice as the immune response 
at early infancy is not developed enough to launch a protective antibody response. 
(Choice E)
Choice A: Tetanus vaccination is not given at birth. The first dose of the diphtheria­
rertussis­tetanus (DPT) series is commonly begun at 2 months of age. In areas 
endemic for tetanus vaccination can be started at 4 to 6 weeks of age. At birth infant's 
immune system is immature and unable to mount a proper immune response against 
the tetanus toxoid. It is also possible that IgG antibodies against tetanus derived from 
mother may bind to tetanus toxoid and inactivate the vaccine. 

Choice B: Post partum antibiotics are not provided to newborns for tetanus 
prophylaxis. They are used manly in cases where group B streptococcal infection or 
colonization has been identified in the mother. They are also used in cases of HIV­
infected mothers in whom anti­ retroviral therapy has been initiated peripartum. 
Choice C: Extension of postpartum hospital stay may help in providing more rapid 
medical treatment to an infant with neonatal tetanus, but it would not reduce the 
incidence of neonatal tetanus.
Choice D: As breast milk is rich in antibodies, early start of postpartum breastfeeding 
is very important in reduction of infectious disease in newborn. However passive 
immunity provided to infants is generally mucosal, because IgA antibodies are mainly 
present in breast secretions. It will not reduce the incidence of neonatal tetanus, 
unless the mother herself is having high titers of IgG antibodies against tetanus. This 
can be ensured by immunization of the mother during pregnancy, so that IgG 
antibodies are able to cross the placenta to reach the fetus.
 
Educational Objective:
Tetanus can be easily prevented by proper immunization with a series of childhood 
vaccinations and a booster immunization every ten years thereafter. An immunized 
mother will be able to pass IgG anti­tetanus antibodies, through the placenta to the 
fetus and provide passive immunity against neonatal tetanus. Vaccination of infant 
just after birth is ineffective because on immature immune system at this age and 
hence the inability to mount an immune response
54
A 20­year­old farm worker presents to the hospital with paroxysmal, painful 
involuntary muscle contractions mainly of the jaw and trunkal muscles. Causative 
organism of this affliction commonly takes the following route in establishing the 
disease.

Answers 
                A. Wound → axons → salivary glands
               B. Wound → motor neuron axons → spinal cord
                C. Food → systemic circulation → peripheral nerves 
                D. Food → systemic circulation → meninges
                E. Fibrinous exudate → systemic circulation → cortical neurons
Explanation:
The patient is likely to be suffering from tetanus, a disease caused by anaerobic spore 
producing gram positive bacteria ­ clostridium tetani. Bacterial spores are commonly 
present in soil and can easily contaminate injured and devitalized tissue. Such tissues 
provides ideal environment for growth of vegetative form of the bacteria because of its 
low oxygen tension. Vegetative form of the bacteria produces a powerful toxin called 
tetanospasmin, which is a potent neurotoxin. After bacterial autolysis, the toxin is 
released and gains access to the motor neuron via the axonal terminal. The toxin then 
travels up the motor neuron by retrograde transport to the spinal cord and medulla. 
Tetanospasmin blocks release of inhibitory neurotransmitters glycine and gamma 
aminobutyric acid (GABA). With diminished inhibition of resting neurons, the motor 
neurons can fire easily. Classic feature of tetanus include stiffness of the jaw due to 
spasmodic contraction of the masseter muscle (trismus) and sustained contraction of 
the facial muscles, producing a bizarre ‘smiling’ appearance (risus sardonicus). In later
stages, muscles of neck and back also show spasmodic contractions leading to 
opisthotonos. Respiratory muscle involvement produces respiratory failure. (Choice B)
Choice A: Wound → neuron axons → salivary glands is the route taken by the rabies 
virus. After its inoculation into a bite­wound from the saliva of a rabid animal, the 
virus is taken up at the motor neuron terminal and travels by retrograde axonal 
transport into the nerves of the spinal cord and from there into the cerebellum, brain­
stem and hippocampus. From here it travels within nerve axons to the tissues of the 
salivary gland.
Choice C: A Food → systemic circulation → peripheral nerve is the route taken by the 
botulinum toxin, which causes food­come botulism.
Choice D: Food → systemic circulation → meninges is the route taken by listeria 
monocytogenes , a gram positive food­borne pathogen that causes meningitis in 
neonates end immunocompromised patients.
Choice E: Fibrinous exudate → systemic circulation → cortical neurons is the route 
that diphtheria toxin takes from the pseudomembranous exudate in oropharynx to the
bloodstream and subsequently to cardiac and cerebral cortical tissues.
Educational Objective:
Clostridium teteni is responsible for tetanus, a toxin­mediated disease that causes 
uncontrolled muscle spasms and respiratory failure. Toxin travels within the motor 
neuron by retrograde transport into the spinal cord where it causes inhibition of 
inhibitory neurotransmitters glycine and gamma aminobutyric acid (GABA). This 
causes uncontrolled firing in motor neurons producing uncontrolled spasms of  the 
muscles.
55
A 7­year­ old boy presents to the hospital with high fever, altered consciousness and 
headaches. History or vomiting is also present. Physical examination reveals neck 
rigidity and a purpuric rash on extremities. His CSF examination shows decreased 
glucose, increased proteins and large number of polymorphs. This infection could have 
been prevented by a vaccine containing
Answers  
                Α. Live attenuated bacteria 
               B. Bacterial capsular polysaccharide
C. Bacterial outer membrane protein 

                D. Inactivated toxin
                E. Heat­killed bacteria
Explanation:
This patient is presenting with classic signs and symptoms of acute bacterial 
meningitis. Common pathogens of acute bacterial meningitis include Str. pneumoniae,
N. meningitides, Str. agalactiae, Listeria monocytugenes, H. influenzae, S. aureus and
E.coli. Less commonly it may be caused by, Borrelia, Leptospira, and T. pallidum.  
Causative organism of acute bacterial meningitis cannot be suspected based on clinical
signs and symptoms alone. However presence of purpuric rash does suggest infection 
by Neisseria meningitides, as in this case. Neisseria meningitides is a gram negative 
diplococcus transmitted by respiratory secretions. Close living conditions (e. g., college 
dormitories. prisons, and military barracks) promote its spread. 
N. meningitidis has several virulence factors that include a polysaccharide capsule, 
(impairs phagocytosis of the bacteria), lipopolysaccaride endotoxin, pili (for attachment
to the respiratory mucosa), and IgA protease (cleaves secretory IgA that would 
otherwise inactivate the pili). Antibodies against the polysaccharide capsule confer 
immunity against N. meningitides. Meningococcal vaccine consists of a single injection
of polyvalent polysaccharide capsular antigens of serogroups, A, C, W­135 and Y. 
Serogroup B antigens are weak immunogens, hence vaccine against this subgroup of 
N. meningitidis is not available. Vaccine provides about 80% to 90% immunity in 
adults. Unless multiple injections are given, the vaccine is ineffective in children < 2 
years old. Immunity lasts for about 5 years. Because immunity provided by the 
vaccine is incomplete and transient, it is only used only in high­risk groups like 
military recruits and college students. (Choice B)
Choice A: The BCG vaccine is used in some parts of the world for protection against M.
tuberculosis contains live­attenuated organisms of a different mycobacterial species. It
is not used in USA. Salmonella typhi also contain live­attenuated bacteria.
Choice C: Borrellia burgdorferi (Lyme disease) vaccine contains recombinant bacterial 
outer surface protein 
Choice D: Vaccine for Clostridium tetani contain inactivated toxin (toxoid).
Choice E: Heat ­killed bacteria are included in B. pertussis, V. cholera and Y. pestis 
vaccines.
Educational Objective:
N. meningitides can cause upper respiratory infection, meningitis and 
meningococcemia Immunity against this bacterium is provided by antibodies against 
their polysaccharide capsules. The meningococcal vaccine contains immunogenic 
capsular polysaccharides from four major subgroups of N. meningitides (serogroups A, 
C, W­135 and Y) and induces production of protective anti­capsular antibodies
 
 
56
A 36­ year­old male is brought to the hospital with a one­week history of headache, 
progressive confusion and occasional seizures. About a year back he developed 
cytomegalovirus esophagitis and two months ago he was hospitalized with 
pneumocystis carinii pneumonia. His CSF findings show moderate increase in protein 
along with pleocytosis. A latex agglutination test is positive for soluble polysaccharide 
antigen of a particular organism. An India ink preparation of CSF allows visualization
of the organism. Organism seen in this preparation would be in form of:
Answers
A. Budding yeast forms

B. Nonseptate hyphae

C. Conidiophores

D. Spherules

E. Sporangium

Explanation:
A recent history of CMV esophagitis and pneumocystis carinii pneumonia is virtually 
diagnostic of HIV infection. These organisms are opportunistic pathogens and 
normally produce disease only in immunocompromised hosts. The patient in question 
is currently having symptoms that are fairly characteristic of meningitis.  His CSF 
changes are also typical of meningitis. In an HIV positive patient, infections of CNS 
are likely to be caused by Cryptococcus neoformans, C.immitis, Histoplasma 
capsulatum, Acanthamoeba, Naeglaria, Toxoplasma, CMV,                 T. pallidum, M. 
tuberculosis and HTLV­1. Cryptococcus neoformans is the commonest fungi producing 
meningitis in HIV infected patients. India Ink preparation for C..neoformans is a 
negative stain, where the capsule of this yeast like organisms is not stained and 
appears as a halo around the organism. Other investigations for diagnosis of 
crptococccal meningitis are latex agglutination or EIA tests for detection of antigens of
this organism. Sensitivity of these tests is around 90%. For study purposes, the 
medically important facts regarding C. neoformans are summarized below. (Choice A)
Choice B: Nonseptate hyphae that branch at wide angles are characteristic of mucor 
and rhizopus species of fungi. These fungi cause infection of the paranasal sinuses in 
immunosuppressed patients.
Choice C: Conidiophores is the branch of mycelium that bears spores. It normally is 
seen in artificial media during culture of the fungus
Choice D: Spherules are found in the tissue form of Coccidioides immitis. This fungus 
causes lung disease and disseminated mycosis, but is not commonly associated ­with 
meningitis.
Choice E: Sporangium is a sac like that produces and contains spores. It is present in 
mold fungi. Cryptococcus occurs only in yeast form and thus does not have sporangia.
Educational Objective:
Cryptococcus neoformans causes meningoencephalitis in HIV positive individuals. The
latex agglutination test detects the polysaccharide capsule antigen of cryptococcus and
is used for diagnosis. India ink staining of the CSF shows round or oval budding yeast 
and is used for rapid provisional diagnosis
57
A patient suffering from Hodgkin’s disease is put on chemotherapy. After few cycles of 
treatment, the patient develops fever. A blood culture is done. It shows growth of 
many Gram positive rods that produce a narrow zone of hemolysis on sheep blood 
agar. The organisms are catalase positive. Which of the following processes is most 
important in eliminating these bacteria from the body?
Answers  
                A. Terminal complement cascade
                B. Activity of major basic protein
               C. Cel1­mediated immunity
     D. Immunoglobulin secretion
                E. Neutrophil oxidative burst 
 
Explanation:
Gram positive beta hemolytic rods that infect humans are commonly, Listeria 
monocytogenes and Arcanobacterium (produces respiratory tract and skin infection 
and are catalase negative). It is a facultative intracellular parasite and the only Gram 
positive bacteria to produce lipopoysacchande (LPS) endotoxin. Listeriae can cause 
some serious diseases like meningitis and septicemia in newborn infants, pregnant 
women (abortions or still birth), and the elderly and immunocompromised patients. It 
rarely causes disease in immunocompetent individuals. 
Resting macrophages when they ingest L. monocytogenes, are unable to kill the 
bacteria, which go on to multiply in the cytoplasm and infect other cells. On the other 
hand, macrophages that have been activated by IFN­γ (secreted by NK cells and T 
cells) are able to destroy the microorganisms. In this way healthy individual, are able 
to get rid of these organisms. However patients whose immune system is compromised
are unable to mount T­cell or NK cell response. (Choice C)
Listeriosis may be seen in neonates or in adults. In neonatal listeriosis, the organism 
is transmitted from the mother to the fetus either transplacentally or during fetal 
passage through the vaginal canal. In both adults and neonates, listeriosis can be 
transmitted by the consumption of contaminated food.

Choice A: Patients with inherited deficiencies of the terminal complement cascade are 
unable to form the membrane attack complex (MAC) and usually are predisposed to 
recurrent infection by pyogenic organisms like Neisseria and pneumococcus.
Choice B: major basic protein is produced by eosinophils and is responsible for killing 
of multicellular parasites
Choice D: X­linked agammaglobulinemia of Bruton is a condition causing a pure defect
of immunoglobulin synthesis and secretion. Patients lack humoral immunity but have 
intact cell­mediated immunity. These patients are therefore at increased risk for 
bacterial infections like Streptococci and Staphylococci but have normal responses to 
infections like Listeria, viruses and fungi.
Choice E: Defect in neutrophil oxidative burst leads to chronic granulomatous disease 
of childhood (CGD). This is an X­linked disorder that causes decreased NADPH 
activity and a failure of the myeloperoxidase system, resulting in an inability to form 
hydrogen peroxide and oxygen free­radicals in concentrations high enough to kill 
catalase­producing bacteria.
Educational Objective:
Intact cell­mediated immunity is essential for elimination of Listeria monocytogenes. 
Neonates up to 3 months of age are especially vulnerable because their cell­mediated 
immune responses are not well developed. Same happens with immunocompromised 
adults. Listeria monocytogenes rarely causes disease in normal healthy adults.
58
A patient is having uncontrollable contraction in his neck muscles. Immediate relief 
that lasts for up to a few weeks is obtained, when a bacterial product is injected locally
into these muscles. This substance is produced by bacteria that also demonstrate:
Answers              
     A. IgG­binding outer membrane protein
                B. Antiphagocytic capsule
                C. Hypervariable pili
               D. Subterminal spore formation 
                E. Intracellular polyphosphate granules
Explanation:
Focal dystonia is a localized uncontrollable muscle contraction that causes pain or 
discomfort, as well as physical deformity in some cases. Some examples of focal 
dystonia are blepharospasm, oromandibular dystonia and contraction of 
sternocleidomastoid muscle called torticollis. Local injection of botulinum toxin type B 
into contracted sternocleidomastoid muscle results in muscle relaxation because the 
toxin prevents presynaptic release of acetylcholine by producing proteolysis of this 
neurotransmitter. The effect is temporary however; because regeneration of the nerve 
terminal occurs eventually (regeneration occurs in about three months). For this 
reason therapeutic botulinum toxin injections must be repeated, when effect of the 
earlier injection begins to wane.
Botulinum toxin can also be used cosmetically to reduce the appearance of glabellar 
and other facial wrinkles. It is also used to relax the lower esophageal sphincter in 
case of esophageal achalasia, to treat muscle spasms of multiple sclerosis and 
Parkinson's disease, and for other conditions resulting from involunatary muscle 
contraction.
Clostridium botulinum is a Gram positive anaerobic bacillus forming subterminal 
spores. It is found in soil and in sea water all over the world. It elaborates one of the 
most potent toxins amongst all bacteria. The powerful neurotoxin is synthesized 
intracellularly and is released after autolysis of the bacteria. (Choice D)

Choice A: Staphyloccus aureus has an IgG­binding outer membrane protein, the 
protein A virulence factor. Protein A binds to the Fc portions of IgG molecule thereby 
preventing opsonisation, phagocytosis and complement fixation.

Choice B: The anti­phagocytic capsule is a primary virulence factor of                      S. 
pneumoniae, H. influenzae and Neissera.
Choice C: Hypervariable pilli are characteristic of Neisseria meningitidis and N. 
gonorrhoeae.
Choice E: Intracellular polyphosphate granules are a characteristic of 
Corynebacterium diptheriae. Granules within the cytoplasm are evident with 
methylene blue staining technique.

Educational Objective:
Clostridia are Gram positive spore­forming anaerobic rods. C. botulinum is the 
bacteria responsible for botulism, a toxin­mediated disease. Local injections of 
botulinum toxin into muscle are used to treat focal dystonias, achalasia and spasms n 
patients of multiple sclerosis.
 
 
59
A group of school children living in a dormitory develop a disease characterized by 
high fever, headache, nausea, vomiting and hemorrhagic skin rash. There blood 
culture shows growth of Gram­negative, kidney­ bean shaped cocci. Which of the 
following microbial components of these bacteria can be related to the morbidity and 
mortality of the disease caused by these organisms?
Answers 
                A. Inner membrane protein
B. Cell wall lipopolysaccharide
                C. Exotoxin
                D. Outer membrane protein
                E. Capsular polysaccharide
Explanation:
The lipooligosaccharide (LOS) of N. meningitidis is analogous to the lipopolysaccharide
(LPS) of other enteric gram negative rods. In case of N. meningitidis, the cell with its 
LOS acts as an endotoxin, which is associated with many of the toxic effects of 
meningococcal disease. Plasma levels of LOS are closely associated with disease 
manifestation and outcome in meningococcal infections.
As with other Gram­negative infections, growth of N. meningitidis during the infection
process causes release of outer membrane vesicles (OMV) containing LOS into the 
bloodstream. The severity of the meningococcal disease has been shown to correlate 
with increasing concentrations of OMV­bound LOS. As in case of LPS produced by 
other gram negative bacteria, LOS causes sepsis by induction of a systemic 
inflammatory response. This occurs due to production of pro­inflammatory cytokines 
like tumor necrosis factor alpha (TNF­α), inteleukin­13 (IL­13), IL­6 and IL­ 8 by 
monocytes, neutrophils and endothelial cells.  Additionally, LOS has been implicated 
as a cause of cutaneous patches and hemorrhagic bullae found in meningococcemia. It 
is also believed to be the cause of bilateral adrenal cortical hemorrhage characteristic 
of the Waterhouse­Friderichsen syndrome. (Choice B)

Choice A: Inner membrane protein along with outer membrane protein of N. 
meningitidis principally act in cellular homeostasis for the bacteria 
Choice C: Exotoxin production is not a known mechanism of pathogenicity of           N. 
meningitidis. Better known exotoxins are the staphylococcal toxic shock syndrome 
toxin, diphtheria toxin, tetanus toxin and pertussis toxin. 
Choice D: Outer membrane protein is not responsible for the production of sepsis in 
meningococcemia. LOS is the causative agent for causation of septic symptoms in 
patients affected by N. meningitidis
Choice E: The capsular polysaccharide of the group C and A serotypes of                 N. 
meningitidis elicit an immune response. Group B serotype is shown to be a poor 
immunogen and is not covered by the meningococcal polysaccharide vaccine. 
Individuals who become colonized with N. meningitidis produce protective antibodies 
directed against the capsular polysaccharide for that particular serotype only.

Educational Objective:
Meningococcal   lipooligosaccharide (LOS) is responsible for many of the toxic effects 
observed in meningitis and meningococcemia. Blood level of LOS correlates with 
disease morbidity and mortality.
60

A 30­year­old male presents to the hospital with complaints of severe headache, 
nausea, vomiting and fever. His cerebrospinal fluid (CSF) findings are as follows: 
 Pressure             260 cm H20

 Glucose                   16 mg/dL

 Protein                    186 mg/dL

 WBC                        560 cells/cmm

o Neutrophils       92%

o Lymphocytes     08%

If a Gram stain is performed on his CSF, it is likely to reveal:
Answers 
A. Motile Gram positive rods

B. Bean­shaped Gram negative cocci in pairs

C. Gram positive cocci in clusters

D. Lancet­shaped Gram positive cocci in pairs

E. Non­motile Gram negative coccobacilli

Explanation:
Symptoms of headache, vomiting, fever, and neck rigidity are very suggestive features 
of meningitis. Meningitis can be infectious, chemical­induced or it may be due to 
infiltration by neoplastic cells. Infective meningitis can be bacterial, viral, fungal or 
parasitic in origin. In bacterial meningitis, CSF analysis typically shows elevated 
neutrophils, decreased glucose, and elevated proteins. The morphology of any observed
bacteria on CSF Gram stain provides an excellent preliminary identification of the 
causative pathogen.
Bacteria commonly implicated in acute meningitis in adults are E.coli, S. pneumoniae,
N. meningitidis, H. influenzae and Staphylococcus aures.  At extremes of life, group B 
streptococci and Listeria monocytogenes are seen in greater number. After 
introduction of vaccination against H .influenzae, Streptococcus pneumoniae has 
become the commonest causative bacteria in adults in USA (about 50% of the cases).  
S. pneumoniae appear as pair of lancet­shaped Gram positive cocci. S. pneumoniae 
meningitis often follows a pulmonary infection or mild upper respiratory infection. 
After gaining access to the blood stream, the bacteria are able to evade phagocytosis 
and complement induced destruction because of their polysaccharide capsule. Once the
bacteria reach the CSF, they rapidly proliferate because of absence of effective 
immune response at this site. Teichoic acid and peptidoglycan of S. pneumoniae induce
inflammation in meninges. Alcoholics, sickle cell anemia patients, asplenic individuals
or those in generally poor health are at risk for meningitis caused by S pneumoniae. 
(Choice D)
 

 
 Choice A:  Listeria monocytogenes has the morphologic appearance of motile Gram 
positive rods. It is a facultative intracellular bacterium; the only Gram positive 
bacteria capable of producing lipopolysaccharide. It produces meningitis at extremes of
age and in immunocompromised individuals.
 Choice B: Bean­shaped Gram negative cocci in pairs is the typical morphology 
observed with N. meningitidis, the second most common cause of meningitis in 
patients less than 60 years of age. N. meningitidis meningitis tends to occur more 
often in individual living in close quarters as in dormitories etc. 
Choice C: Gram positive cocci in clusters are the typical morphology observed with 
Staphylococcus species. Staphylococci are an unusual cause of meningitis. 
Staphylococcal meningitis occurs most often in neurosurgical patients due to direct 
access to meninges.
 Choice E:  H. influenzae is a Gram negative coccobacillus. Infection with this 
organism is becoming less common because of vaccination with the Hib capsule 
vaccine. Previously, H. influenzae was a major cause of meningitis in very young 
children, with a mortality rate of about 5%. 
 
Educational Objective:
Bacterial meningitis causes an increase in CSF neutrophil count and protein 
concentration along with decrease in glucose. Streptococcus pneumoniae is a leading 
cause of community­acquired pneumonia. S. pneumoniae appear in pair as lancet 
shaped Gram positive cocci
61
A 45­ year­old male, who recently underwent renal transplantation and is on 
immunosuppressive drugs, is brought to the hospital with complaints of headache, 
fever and altered consciousness. His CSF findings are: presence of pleocytosis, normal 
glucose levels and presence of Gram positive rods with tumbling motility. What is the 
likely route of infection in this patient? 
Answers
A. Arthropod bite

B. Unprotected sex

C. Respiratory droplets infection

D. Needle stick injury

E. Contaminated food 

Explanation:
Development of meningitis in an immunosuppressed individual is most often due to 
infection by Listeria monocytogenes. In adults, listeriosis occurs almost exclusively in 
the immunocompromised persons. The bacteria commonly gains access to the blood 
stream following ingestion of contaminated food. Unpasteurized milk and milk 
products, undercooked meat and unwashed raw vegetables are common culprits. It 
also produces amnionitis in pregnant women that can result in abortion or stillbirth.
L. monocytogenes is an opportunistic agent and a facultative intracellular parasite 
that grows within macrophages in immunocompromised human hosts. It is able to 
multiply at 4 C, a unique feature that laboratories exploit when culturing the 
organism, a process called cold enrichment. Macrophages and T lymphocytes are the key
defense mechanism against Listeria (Choice E)
Choice A: Arthropod bites transmit leishmaniasis (sandfly bite), malaria (anopheles 
mosquito bite), Chagas disease (tsetse fly bite) and Borrelia burgdorferi (Lyme 
disease).
Choice B: Unprotected sex is responsible for transmission of sexually transmitted 
diseases like HIV, hepatitis B and C, gonorrhea, chlamydia, syphilis, LGV, human 
papilloma virus infection and others.
Choice C:  N. meningitidis, H. influenzae, respiratory syncytial virus, M. tuberculosis, 
and many other organisms are transmitted via respiratory droplets. However Listeria 
monocytogenes is not transmitted through respiratory droplets.
 Choice D: Needle stick injury can transmit hepatitis B, hepatitis C, HIV as well as 
some other infections
 
 Educational Objective:
 Listeriosis is most commonly transmitted through contaminated food. It causes 
sepsis, meningitis and brain stem and spinal cord abscesses in immunocompromised 
adults. However in about 30% of cases no risk factor is recognized. Listeria 
monocytogenes can also cause neonatal meningitis, it being transmitted 
transplacentally or during vaginal passage. Infection during pregnancy may cause 
abortion or stillbirth. Listeria grows well in cold temperatures and thus can 
contaminate refrigerated food.  Listeria is a gram positive rod with a unique tumbling 
motility.
62

A 30­year­old male presents to the hospital with 6 months history of productive cough,
fatigue, malaise and weight loss. Chest radiography shows presence of pulmonary 
infiltrate in the lower lobe of the right lung. Potassium hydroxide preparation of his 
sputum reveals the following.

What is the most likely diagnosis?

Answers:
A. Histoplasma  capsulatum
B. Aspergillus  fumigatus
C. Candida albicans
D. Cryptococcus neoformans 
E. Blastomyces dermatitidis
F. Coccidioides  immitis

Explanation:
The image above shows encapsulated thick walled yeast with single, broad­based 
budding. This is the typical appearance of Blastomyces dermatitidis, a fungus endemic
to the Great Lakes, and Ohio and Mississippi River regions. It is present in soil end 
rotten organic matter. Blastomyces is a dimorphic fungus, meaning it assumes a 
different form at different temperatures.  The mold form (branching hyphae) 
predominates in the environment having average temperatures of 25­30°C. In human 
body (37­40°C), it assures the yeast form (single cells). Yeast forms are usually 10 to 
12 μm in size; they have thick refractile walls and centrally retracted cytoplasm.

Infection occurs by inhalation of aerosolized fungus from the environment. In the 
lungs, Blastomyces assumes the yeast form. It multiplies and induces a 
granulomatous response. In about 50% of immunocompetent individuals, 
blastomycosis remains asymptomatic. In others, it may present as a flu­like illness or 
pneumonia. Pulmonary blastomycosis is diagnosed by PAP staining of KOH 
preparation of the sputum, which demonstrates the typical morphology of this fungus. 
In immunocompromised patients, blastomycosis can cause disseminated disease and 
patients will experience systemic symptoms, cutaneous and skeletal manifestations. 
CNS and genitourinary systems are involved less frequently. Histology of lesions 
caused by B, dermatitidis is usually a mixture of acute and chronic granulomatous 
reaction. (Choice E)

Choice A: Histoplasma shows presence of yeast forms that measure 2­4 μm in size. 
Yeasts of histoplasma divide by budding which distinguishes it from Penicillium 
marneffei. In blastomyces yeast form, have sizes ranging from 2 – 10 μm.

Choice B: Aspergillus infection usually shows septate hyphae that have parallel walls 
and branching at a 45 degrees angle.

Choice C: C. albicans shows presence of pseudohyphae and budding and non­budding 
yeast cells.

Choice D: C. neoformans produces disease in immunocompromised individuals. In 
contrast to blastomyces, it forms narrow based buds. It measures 3 – 10 μ m in size. 
Cryptococcosis has a thick polysaccharide capsule that appears clear with India ink 
staining and stains red with mucicarmine. 

Choice F: C. immitis is also a dimorphic fungus that shows presence of spherules that 
contain endospores.
 

Educational Objective:
Blastomyces dermatitidis a dimorphic fungus that is seen in tissue as round yeasts 
with doubly refractive walls and broad based budding from a single pole on the mother
cell.. The lungs are the primary site of involvement. Infection by this fungus occurs 
mainly in central and mid­western states of USA and eastern Canada.
63
A researcher finds that Streptococcus pneumoniae injected intraperitoneally in mice 
results in death of the animals. Antibodies having which of the following effects on the
bacterium would prevent this outcome.
          
A. Causing loss of cytotoxicity
B. Decreasing motility of the bacteria
C. Impairing adhesion of bacteria to epithelial cells
D. Causing swelling  of bacterial capsule
E. Inhibiting hemolysis 

Explanation:

Pathogenic species of S. pneumoniae is commonly called pneumococcus. It is found as 
normal flora in oropharynx in many individuals. It may produce disease of the middle 
ear, paranasal sinuses, lung, joints, endocardium and meninges to produce disease. 
The capsular polysaccharide is the major virulence factor. Strains with a thick mucoid 
capsule are especially virulent. It has antiphagocytic properties.

When viewed under a microscope, the capsule swells when specific anti­capsular anti 
bodies are added (Choice D) an effect known as the "quellung reaction". This reaction   
can be used to identify S. pneumonia and to serotype the isolate. The capsule can also 
be visualized by addition of India ink to a suspension of bacteria. Capsular region of 
the bacterium appears as a clear halo.

Choice A: Pneumococcus can destroy ciliated epithelial cells via the toxin pneumolysin.
This protein also helps the pneumococcus survive phagocytosis by suppressing the 
phagocytic oxidative burst. Pneumolysin is not the target of any specific antibody nor 
is it a major virulence factor.

Choice B: S. pneumoniae is a nonmotile organism, hence antimotilty effects are f no 
relevance.

Choice C: The adhesion of S. puemoniae occurs by specific interaction of bacterial 
surface adhesions with epithelial cell receptors. However, apthogeencity of bacteria is 
dependant on anti­phagocytic property of the bacterium and not on its adhesion 
molecules.
Choice E: S. pneumoniae produces alpha hemolysis on blood agar, meaning that it 
produces incomplete hemolysis. Neutralization of this property would not be of any 
benefit for controlling virulence of these bacteria
Educational Objerctiva:
S. pneumoniae expresses a polysaccharide capsule that inhibits phagocytosis by 
macrophages .and polymorphonuclear leukocytes. It is the primary virulence factor, 
without which S. pneumoniae cannot cause disease.
64
A 74 ­year­old man develops high fevers, chest pain and productive cough. Sputum 
microscopy shows many lancet­shaped Gram­positive cocci in pairs. The patient dies 
despite adequate antibiotic treatment. A vaccine containing which of the following 
components could have prevented this outcome.

Answers

A. Killed bacteria
B. Recombinant surface protein 
C. Inactivated toxin
D. Live attenuated bacteria 
E. Synthetic nucleic acid vaccines
F. Capsular polysaccharides 

Explanation:
There are more than 90 serotypes of S.pneumoniae based on the variation in capsular 
polysaccharide. Diversity of serotypes makes vaccine development a complex task. The
vaccine currently licensed for use has a mixture of 23 polysaccharide serotypes. It 
offers protection against 90% of isolates of pneumococci. These single­dose vaccines are
recommended for all adults over the age of 65 years and for other patients at high risk 
for pneumococcal sepsis (e.g. asplenic patents, patients with chronic obstructive 
pulmonary disease and immunosuppressed patients). The vaccine has an efficacy of 
about 85% in persons younger than 55 years. But the efficacy decreases as the person 
ages; persons in their 80’s have only 50% protection after vaccination. This vaccine is 
also not efficacious in infants younger than 2 years. For them a hepatavalent protein 
polysaccharide conjugate (instead of the usual polysaccharide vaccine) vaccine has 
been introduced. It has an efficacy of around 90% in this age group. Since vaccine 
efficacy is not complete, antibiotic prophylaxis is used by many in high­risk group 
patients (Choice F.)

Choice A: Killed bacterial vaccines include those for anthrax, cholera, pertussis and 
plague. Killed vaccines usually require multiple inoculations in order to induce 
immunity, but they do not possess the risk of reactivation. This vaccination method is 
also more commonly used in oral vaccines, including the hepatitis A, influenza, 
rubella, rabies and Salk polio vaccines.

Choice B: The Hepatitis B vaccine is a recombinant surface protein vaccine.
Choice C: Inactivated toxin vaccines include diptheria and tetanus vaccines. 
Inactivated toxin vaccines allow the body to immunologically recognize the toxin and 
inactivate it by antibody binding.
Choice D: Live attenuated bacterial vaccine includes the BCG vaccine, used outside of 
the United States to immunize against tuberculosis and the typhoid vaccine against 
Salmonella typhi. Live attenuated vaccines are vey effective in inducing immunity as 
the organisms are actively growing in inoculated host. The risk associated with these 
vaccines is the live organisms potential to revert to the virulent state. 

Choice E: Synthetic oligopeptide vaccines present an experimental mode of 
vaccination. The vaccines may be DNA or RNA vaccines. In this, a synthetic nucleic 
acid with help of a plasmid vector is introduced into the body, where it integrates into 
the genome. Expression of the integrated DNA sequence results in a protein product 
that stimulates an immune respose, either humoral or cellular. The concept is still 
experimental, but prospects of multiple advantages of this approach are seductive. 

Educational Objective:
The pneumococcal polysaccharide vaccine is recommended for all adults over 65 years 
of age and for patients with COPD, asplenia, or immunosuppression. Vaccination does 
not completely prevent pneumonia, as this vaccine contains antigen from only 23 out 
of more than 80 different capsular serotypes know. The adult pneumococcal vaccine is 
an unconjugated polysaccharide vaccine that does not stimulate a T­helper cell 
response, unlike the new vaccine approved for infants 
65

A new that blocks bacterial peptidoglycan cross­linking has been synthesized. There is
great enthusiasm for this drug as it is also resistant to degradation by bacterial 
enzymes. Which of the following bacteria is likely to be resistant to this new drug?

Answers

A. Mycoplasma Hominis
B. Pasteurella multocida
C. Borrelia   burgdorferi
D. Helicobacter pylori 
E. Actinomycosis israelii
F. Treponema pallidum 

Explanation:

The new antibiotic inhibits peptidoglycan cross­linking. This moiety is found in cell 
wall of both Gram positive and Gram­negative bacteria. The cell wall is essential for 
survival of bacteria as it acts as a permeability barrier and protects them from 
destruction by osmotic stresses. In absence of proper cross­linking of peptidoglycan, 
cell wall synthesis becomes defective and the bacteria are killed. Thus, the new agent 
would be effective against those microorganisms in which peptidoglycan are a 
constituent.
Mycoplasmas are the smallest prokaryotes that can survive outside of a cell. 
Organisms of Mycoplasma genus that include pathogens like M. pneumoniae,               
M. hominis and Ureaplasma urealyticum lack cell walls. They have only a single 
bilayered phospholipid membrane. Their cell membrane contains cholesterol, just as 
human cell membranes do. . Thus, cell wall synthesis inhibitors such as penicillins, 
cephalosporins, carbapenems, vancomycin and this new agent would be ineffective 
against these organisms. Antibiotics that are active against Mycoplasma genus 
include, anti­ribosomal agents like macrolides and tetracycline. (Choice A)

Choice B: Pasteurella multocida is a Gram­negative organism well known for causing 
wound infections following cat bites. P. multocida can be effectively treated with 
penicillin because penicillin disrupts the very thin but essential peptidoglycan­
containing cell wall of these organisms.
Choice C and F: Borrelia burgdorferi and Treponema pallidum are disease­causing 
spirochetes that can be effectively killed by this new agent because their cell wall has 
peptidoglycan.

Choice D: Helicobacter pylori are Gram­negative helical organisms that are closely 
related to the Campylobacter genus. Multiple agents are required for effective 
eradication of this infection.

Choice E: Actinomyces israelii is a gram­positive fungus­like bacterium that has a 
thick peptidoglycan cell wall. Hence, the new agent would be able to act against this 
organism.

Educational Objective:
All organisms in the Mycoplasma genus lack peptidoglycan cell walls and are therefore
resistant to agents whose mode of action is to interfere with peptidoglycan containing 
cell wall. Hence, agents like penicillin, cephalosporin, carbapenems and Vancomycin 
are ineffective. Mycoplasma infection can be treated with anti­ribosomal agents like 
tetracycline and erythromycin.
66
An outbreak of respiratory infection has occurred in a group of schoolchildren living in 
a dormitory. Children have mild clinical symptoms.  However their chest x­ray shows 
infiltrates more severe than would be expected from their clinical status. Culture of 
their sputum on PPLO medium (a medium enriched with cholesterol and animal 
proteins) allows growth of the causative organism. Which of the following pathogen 
may be responsible for this disease outbreak? 

Answers

A. Coxiella burnetii 
B. Streptococcus pneumoniae
C. Klebsilla pneumoniae
D. Haemophilus influenzae
E. Mycoplasma pneumoniae
F. Legionella pneumophilia

Explanation:
These schoolchildren are suffering from atypical pneumoniae, a condition generally 
caused by Mycoplasma pneumoniae. It was so called because historically, the 
pneumoniae could not be attributed to any known bacterial pathogen. It is believed 
that up to one sixth of all pneumoniae are caused by this agent. In military recruits, it 
is responsible for about half of the cases. Although it causes infection more often in 
children, its severity is more in older individuals.  Patients with atypical pneumonia 
experience a chronic dry nagging cough and low­grade fever. The telltale sign is a 
chest x­ray that looks much worse than the patient’s clinical manifestations. Another 
clue to the presence of this organism as a causative agent is that it can be cultured 
only in a medium enriched with a source of animal protein and serum that supplies 
sterols along with saturated and unsaturated fatty acids. (Choice E). Cholesterol is 
needed for growth on artificial medium because their cell membrane is composed of a 
single cholesterol­rich phospholipid bilayer. These organisms completely lack a 
peptidoglycan cell wall, cell envelope or capsule (Fungi also incorporate sterols into 
their cell membranes, but fungi use the sterol ergosterol).

Choice A: C. burnetii is the etiologic agent of Q­fever. Q­fever is a mild pneumonia­like
illness that results from inhalation of the C. burnetii spores that commonly 
contaminate animal hides. It is an obligate intracellular parasite; hence, it requires a 
cell culture in order to grow.
Choice B: Streptococcus pneumoniae can produce lobar pneumoniae. This organism 
can grow well on standard unenriched blood agar.

Choice C: Klebsilla pneumoniae is a common cause of pneumonia in debilitated, 
hospitalized, and alcoholic individuals. Patients with this infection often expectorate 
jelly like sputum (currant jelly sputum). K pneumoniae can be grown on standard agar
but often MacConkey agar is preferred because it contains bile that inhibits the 
growth of contaminant organisms.

Choice D:  H. influenzae requires chocolate agar (lysed sheep blood agar heated to 70 
­80 degree centigrade) supplemented with factor X  and and a source of NAD in order 
to grow.

Choice F: Legionella pneumophila requires an L­ cysteine supplemented agar to grow.

Educational Objective:
Mycoplasma pneumonia is the causative agent of atypical pneumonia; a condition 
characterized by a nagging nonproductive cough, low­grade fever and malaise. Often 
the chest X­ray suggests a severe pneumonia, even though the patient appears 
relatively well. Mycoplasma species require cholesterol supplementation in order to 
grow on artificial media.
67
A 50­ year­old male is brought to the ER with history of recent onset of high­grade 
fever, confusion, headache and cough with little watery sputum. He has a chronic 
smoker and is suffering from chronic bronchitis. His x­ray chest shows lobar 
consolidation. His sputum microscopy after Gram staining shows large number of 
neutrophils, but bacteria are not visualized. Usual bacterial pathogens could not be 
isolated from his sputum sample that was obtained with much difficulty. What 
microorganism can produce this clinical picture?

Answers 

A. Legionella pneumophila
B. Klebsilla pneumoniae
C. Mycobacterium kansasii
D. Mycoplasma pneumoniae 
E. Streptococcus pyogenes  
F. Coccidiodes  immitis 

Explanation:
The patient appears to be having atypical pneumonia. Atypical pneumonia is usually 
caused by Chlamydia pneumoniae, C. psittaci, M. pneumoniae, C .burnetti, Legionalla 
pneumophilia and some viruses. The classic presentation of Legionnaires’ disease 
includes very high fever accompanied by diarrhea, confusion, and cough causing chest 
pain. The disease usually occurs in a smoker or in an immunocompromised individual 
Legionella pneumophilia is one of the most common causes of community­acquired 
pneumonia. Diagnosis can be difficult because the signs and symptoms are not 
specific, but this disease should be suspected in a patient with radiographic evidence 
of pneumonia with high fever and accompanying gastrointestinal symptoms such as 
diarrhea. Also, there is relatively nonproductive cough and the sputum is usually non­
purulent. As compared to other causes of atypical pneumonia, the clinical course of 
Legionnaires’ disease is more severe, with mortality ranging from 10% in general 
population to about 80% in immunosuppressed patients. Acquiring sputum is difficult 
and frequently unreliable with few or no bacteria seen on Gram staining. The 
organisms can be visualized by Dieterle’s silver impregnation stain. Diagnosis is most 
commonly made by testing for Legionella antigen in urine. Legionella is a common 
contaminant of water and can be spread by inhalation of aerosolized water from 
natural water sources, air conditioners and other water­based cooling systems, as well 
as tap water used in the healthcare setting (Choice A).
Choice B: K. pneumoniae is a Gram­negative rod and a major cause of nosocomial 
pneumonia as well as pneumonia in alcoholic or otherwise debilitated patients. It 
would reveal Gram­negative rods on sputum microscopy.

Choice C: Mycobacterium kansasii is an atypical mycobacterium that can cause 
disease similar to that caused by M. tuberculosis. M. kansasii is an acid­fast rod that 
is similar to L pneumophila in that it is a water contaminant. It is an infrequent cause
of contamination of municipal drinking water systems.

Choice D: Mycoplasma pneumonia is another etiologic agent of atypical pneumonia. 
This is an illness that predominantly affects young adults causing a tracheobronchitis 
or pneumonia. Patients often have a mild fever and mild transient anemia as well as a
cough. Chest radiograph will give the impression of a pneumonia that is much more 
severe than what the patient appears clinically. These organisms also do not stain on 
Gram stain because they do not have a cell wall

Choice E: S. pyogenes is the etiologic agent of streptococcal pharyngitis, impetigo and 
abscess.

Choice F: Coccidiodes immitis is a fungal organism that is frequently an asymptomatic
infection, but it can also cause severe disseminated disease and fungemia in 
immunocompromised patients

Educational Objective:
Legionella pneumoniae causes Legionnaires' disease. Legionnaires' disease is 
characterized by a propensity to produce manifestations in chronic smokers and 
immunocompromised individuals. The disease results in high­grade fever and lobar 
consolidation of lungs with non­productive cough. Sputum if any is usually watery in 
nature. The disease carries a high mortality if not treated promptly. 
68
Increased incidence of pneumonia is seen in residents of a health care faculty for 
elderly persons. The microorganism isolated from these cases can be visualized by 
immunofluorescent microscopy. It also demonstrates slow growth on complex media 
such as buffered charcoal­yeast extract agar (BYCE media) supplemented with 
cysteine, ferric salt and α­ketoglutarate. What can be the possible cause of infection by
these microorganisms?

Answers:

A. Widespread use of antimicrobial agents
B. Infection occuring among hospital staff (nosocomial infection)
C. Contamination of the hospital water system
D. Failed sterilization of mechanical ventilators  
E. Poor isolation of infected patients
F. Widespread use of intravascular devices

Explanation:
The cultural characteristic described above is those of Legionella pneumophila. The 
organism requires special culture media to be grown in vitro. It has exacting 
nutritional requirements and grows best on BYCE media. They also grow better in 
CO2 enriched environment. Colonies grow slowly and will appear usually after 5 days.
L. pneumophila is found usually in water bodies like lakes and streams. It can survive 
for long periods in refrigerated water and it proliferates rapidly in water that is 
stagnant, scaly and has lot of sedimentary deposits. 
Outbreaks of L pneumonia have been associated with inhalation of aerosolized water 
contaminated with these organisms. Contaminated water used to humidify air filters 
in different settings can lead to infection. Tap water used in patient care or to 
humidify air during mechanical ventilation is also a possible mode of transmission of 
infection. (Choice C)

Choice A: Widespread use of antimicrobial agents leads to multi­drug resistant 
organisms, but does not predispose to the development of pneumonia itself.

Choice B: Nosocomial mode of spread is a frequent cause of methicillin­resistant S. 
aureus infection. Health care workers can be asymptomatic nasal carriers of this 
organism. Unwittingly they pass this organism to their patients resulting in wound 
and intravenous catheter­related infections.

Choice D: Failed sterilization of mechanical ventilators can lead to nosocomial 
pneumonia with K. pneumoniae, Acinetobacter and Pseudomonas. A large proportion 
of nosocomial pneumonias occur in intubated patient.

Choice E: Poor isolation of the infected patients can result in the spread of many 
infectious diseases in the hospital, but L .pneumophila is not acquired usually by this 
route.

Choice F: Widespread use of intravascular devices does carry increased risk of 
bactericidal and sepsis but not of legionellar pneumonia.

Educational Objective:
L. pneumophila commonly contaminates natural water bodies. Municipal water, 
humidification systems, air conditioning and water­based cooling systems are the 
usual culprits. The organism is inhaled in aerosolized water and establishes infection 
by the pulmonary route. 
69
A 78­year­ male comes to physician's office complaining of fever of sudden onset along 
with headache, muscle pain, malaise and cough. Two of his family members, who live 
with him also, had similar complaints a few days back, but they recovered with 
symptomatic treatment, within 3 to 4 days. Presently, the patient shows rise in 
temperature and congested throat without purulent exudates. The patient is advised 
conservative treatment. Few days later, this patient is admitted to the hospital with 
complaints of progressive dyspnea, chest pain, and productive cough. What is the 
likely causative organism in this case?

Answers

A. Cytomegalovirus
B. Listeria monocytogenes
C. Staphyloccus aureus
D. Klebsiella pneumoniae
E. Pneumocystis carinii

Explanation:
This patient's present signs and symptoms along with history of short lasting illness of
similar nature in other members of his household are consistent with influenza. 
Influenza virus belongs to Orthomyxovirus family of viruses. It is comprised of 
Influenza A, B and C group of viruses. In classical influenza, the illness is 
characterized by sudden onset of symptoms like chills, fever, headache and myalgia. 
Both upper and lower respiratory tract infections can be present. The disease usually 
lasts from 3 to 5 days.
Subsets of patients stricken by influenza go on to develop secondary bacterial 
pneumonia characterized by recurrent fever, dyspnea and productive cough. The 
elderly are particularly at risk for this complication. This produces a classic biphasic 
pattern of fever. In elderly, severe infection and sudden death may occur especially if 
there is an underlying disease such as cardiovascular disorder or COPD. Physical 
examination and chest radiograph demonstrate pulmonary consolidation. This 
vulnerability to secondary infection is because of virus­induced damage to the 
mucociliary clearance mechanisms of the respiratory epithelium. Pathogens mostly 
responsible for secondary bacterial pneumonia are Streptococcus pneumoniae and 
Staphylococcus aureus. (Choice C)Immunity to influenza is type and subtype specific 
ad mediated mostly by local IgA antibody in the mucosa. However, influenza viruses 
are well known for sudden change in their antigenic structure (antigenic shift).
Choice A: Cytomegalovirus may cause pneumonia, particularly in 
immunocompromised persons.

Choice B: Listeria monocytogenes is an occasional cause of septicemia and purulent 
meningitis in neonates and immunosuppressed individuals

Choice D: Klebsilla pneumoniae is most commonly responsible for nosocomial urinary 
tract infection and pneumonia in asthmatics and IV drug abusers. 

Choice E: P. carinii infection occurs most often in immunocompromised individuals 
because of inhalation of aerosolized droplets of contaminated water.

Educational Objective:
Patients older than 65 years of age are particularly prone to develop secondary 
bacterial pneumonia after influenza infection. The pathogens commonly responsible 
for secondary pneumonia are S. pneumoniae and Staph. aureus

 
70
In an experiment, it is seen that certain strains of Streptococcus pneumoniae are non­
virulent However, when grown on culture media alongside a virulent strain of S. 
pneumoniae, the non­virulent strains become virulent. What is the likely mechanism 
of this change in character of the microorganism?

Answers

A. Uptake of chromosomal fragment from media
B. Pilus­mediated transfer
C. Spontaneous mutation
D. Transposon­mediated DNA transfer
E. Phage­mediated DNA transfer

Explanation:
Certain strains of S pneumoniae express capsular polysaccharides that inhibit 
phagocytosis. Strains lacking capsule are not pathogenic. S. pneumoniae is able to 
undergo transformation, a process involving uptake and expression of chromosomal 
fragments from the environment, when neighboring bacteria die and lyses (Choice B). 
Bacteria capable taking up exogenous DNA and undergoing transformation are said to
be competent. Streptococcus, H. influenzae and Bacillus are some of the bacteria that 
are capable of transformation. Through this process, non­virulent, non­capsule 
forming strains of S pneumoniae can acquire the genetic material that codes for the 
capsule and gain virulence. The process of uptake of exogenous DNA involves certain 
alterations in the cell wall of the bacteria so that The DNA can enter the cell. After 
entering the cell, foreign DNA needs to integrate with the host DNA. For this to 
happen, the foreign DNA should be of short length and have close homology with the 
host DNA. (Choice A)

Choice B: Pilus­mediated DNA transfer is called conjugation. We now know that most 
bacteria are capable of this process. To initiate conjugation, the donor bacterium must 
produce a sex pilus, which creates a direct connect with the receiving bacterium. Only 
bacteria with genetic sequences coding for conjugative ability (e.g.plasmids) can 
initiate conjugation.

Choice C: A mutation is a change in the nucleotide sequence of a gene. Mutational 
changes may be spontaneous or induced. Changes in the nucleotide base sequence can 
cause changes in the transcribed mRNA base sequence, potentially altering the amino 
acid sequence of the protein product. Through this mechanism, a bacterium may begin
to form proteins with potentially useful functions to aid in its survival.

Choice D: Transposon­mediated DNA transfer is a mechanism by which DNA from 
plasmids or phage can be incorporated into the host bacterial DNA genome. Genetic 
material can be moved from one position to another within the genome or DNA can be 
removed from the gene and placed onto a plasmid. The location of a gene in the 
genome is important because it determines its proximity to a promoter or a suppressor
region.

Choice E: Bacteriophage (virus} mediated transfer of DNA from one bacterial cell to 
another is called transduction. While replicating within the infected host bacterium, 
the phage may incorporate host bacterial DNA. When the phage is subsequently 
released and it infects another bacterium, it transfers both phage DNA and 
incorporated bacterial DNA into the newly infected cell. It is a method commonly 
responsible for acquisition of antibiotic resistance by bacteria.

Educational Objective:
S. pneumoniae is able to undergo transformation, which is the uptake and expression 
of chromosomal fragments from the environment made available when another 
bacterial cell dies and undergoes lysis.
71
From a 15­year old boy suffering from ‘flu like’ illness, naked RNA molecules are 
isolated. When these RNA molecules are introduced into cells derived from respiratory
tract, viral genome replication and viral protein synthesis is seen. The patient from 
whom initial RNA molecules were isolated, is probably suffering from which of the 
following infections?

Answers
A. Rota virus 
B. Influenza virus type A
C. Rhinovirus type B
D. Respiratory syncytial virus
E. Human immunodeficiency virus type 1

Explanation;
For naked (nonenveloped) RNA molecules to induce viral protein synthesis in host 
cells, it must act as mRNA capable of using host's intracellular machinery for 
translation. This is possible when the viral RNA molecule is single­stranded and 
positive sense (SS+). By positive sense, we mean that viral RNA should be of same 
polarity as host mRNA.  Such viruses, as they code for all the proteins they require for 
replication, are able to reproduce on their own in host cell cytoplasm. In effect, it 
implies that viral RNA extracted from the virion is infectious on its own. Among the 
viruses listed, only Rhinovirus type B contains SS+ RNA. (Choice C). Generally 
speaking, naked SS+ RNA molecules are infectious, whereas naked single­stranded 
negative sense 
(SS–) RNA molecules and the naked double­stranded RNA molecules are not.
Some other pathogenic, nonenveloped, single stranded, positive sense RNA viruses 
belong to Picorna virus family (e.g. poliovirus, Coxsackie virus, Hepatitis A virus and 
foot and mouth disease virus) and Caliciviridae family (Norwalk agent and Sopporo 
virus)

Choice A: Rotavirus contains double­stranded enveloped RNA. It therefore is incapable
of inducing viral protein synthesis in a host cell without the help of a specific viral 
RNA polymerase.  

Choice B and D: Influenza A is an orthomyxovirus and Respiratory syncytial virus 
(RSV) is a paramyxovirus that contains enveloped SS – RNA. In order for this virus to 
replicate in a host cell, an RNA­dependent RNA polymerase is needed.
Choice E: HIV is a retrovirus that contains DS+ RNA packaged with reverse 
transcriptase (an RNA­dependent DNA polymerase).

Educational Objective:
For a naked RNA molecule to induce viral protein synthesis in a host cell, it must be 
able to act directly as mRNA using the host cellular machinery fro replication. Thus, 
naked viruses that contain single­stranded positive­sense RNA molecule can be 
infectious on their own. 
72
A 5­year­old boy is found to have high serum levels of antibodies against polyribitol  
ribose phosphate (PRP). This boy is not likely to suffer from a particular disease 
caused by a microbe. What is the disease?

Answers    

A. Malignant pustule
B. Pyelonephritis
C. Epigottitis
D. Miliary tuberculosis
E. Rheumatic fever
F. Aseptic meningitis

Explanation:
Epiglottitis is almost exclusively caused by  Haemopilus influenzae type b and was 
commonly seen in children between 2 and 7 years old prior to introduction of the 
polysaccharide protein conjugate Hib vaccine in the late 1980s. H. influenzae was also 
a common cause of meningitis prior to this vaccination.
Epiglottitis is usually an acute infection, with abrupt onset of obstructive laryngeal 
edema. Patients present with symptoms of acute fever, inspiratory stridor, drooling, 
dysphagia, dysphonia and positive thumb sign on lateral cervical x­ray (due to the 
edematous epiglottis). The disease can be life threatening especially in children and 
occurs due to intense cellulitis involving posterior part of the tongue and the epiglottis 
that causes obstruction to airflow. 
Invasive infection by these bacteria is usually caused by encapsulated type b strain of 
H. influenzae. The capsule of H. influenzae is immunogenic and is a polymer of 
ribosyl,  ribitol and phosphate. It is referred to as polyribose­ribitol­phosphate (PRP). 
Capsule is also the main virulence factor of these bacteria. Other virulence factors are 
fimbriae, IgA proteases and outer membrane components. Antibodies to PRP facilitate 
opsonization and complement­dependent phagocytosis of bacteria. (Choice C)
To effectively  prevent infection  with  H. influenzae type b individuals must mount an 
antibody response against the capsular material. The rarity of infection in first two 
months of life is due to presence of maternally derived antibodies. The first dose of H. 
influenzae type b vaccine is given at 2 month of age.

Choice A: Malignant pustule  is a painless ulcer with black eschar and local edema. It  
is caused by Bacillus anthracis. 
Choice B: Pyelonephritis is most commonly caused by E. coli; both in adults as well as 
children and accounts for over 90%  of cases in some studies.

Choice D: Miliary tuberculosis  is caused by Mycobacterium  tuberculosis. Their cell 
wall contains several complex lipids  (mycolic acids) 

Choice E: Rheumatic fever is sequelae of untreated group A Streptococcal pharyngitis. 

Choice F: Aseptic meningitis is usually caused by viruses and certain bacteria that 
cannot be cultured on routine culture media. H. influenzae does not belong to this 
group.

Educational Objective:
The H. influenzae type b (Hib) vaccine is composed of polyribosyl­ribitol­phosphate 
(PRP), a component of the Hib capsule. It usually is conjugated with diphtheria or 
tetanus toxoid. Immune activation with antibody production and generation of 
memory T­lymphocytes against PRP provides lasting immunity against Hib in 
children as young as 2 months old.
73
A 10­month­ old child presents initially in outpatient clinic with fever, running nose 
and sore throat. Two days later, he is brought to the emergency department with 
complaints of persistent fever, dry brassy cough and difficulty in breathing. History of 
a single episode of seizures is also present. Physical examination shows presence of 
stridor and mild cyanosis. Which of the following pathogens can be responsible for this
type of clinical picture?

Answers
A. Calicivirus
B. Rhinovirus 
C. Togavirus 
D. Parvovirus
E. Paramyxovirus

Explanation:
The child presented initially with features of upper respiratory tract infection (URTI). 
Most URTIs  are caused by viruses. Bacterial etiology is present in about15 ­ 20% of 
childhood cases and in 5 ­10% of adulthood cases of pharyngitis. Viruses responsible 
for URTI are rhinovirus, corona virus, parainfluenza virus, respiratory syncytial virus,
influenza, adeno and metapneumovirus.

If a child who has been suffering from URTI develops a brassy cough with breathing 
difficulty, then the possibility of acute laryngotracheobronchitis (croup) should be 
considered. It usually develops 1 to 2 days after the onset of URTI. The dyspnea 
associated with croup occurs when inflamed subglottic tissue obstructs the upper 
airway. This manifests as inspiratory stridor. In croup, neck x­ray (AP view) shows 
subglottic edema (hourglass sign). Croup can be caused by all the viruses mentioned 
above; however, parainfluenza type 1 virus is the commonest cause. Parainfluenza 
virus belongs to paramyxoviridae family (Choice E).

Choice A: Calicivirus are responsible for viral gastroenteritis

Choice B: Rhinovirus is the most common viral cause of upper respiratory infections 
but is an uncommon cause of viral croup

Choice C: Togaviruses are responsible for rubella, Eastern, and Western equine 
encephalitis.

Choice D: Parvovirus is responsible for aplastic crises in sickle cell anemia, erythema 
infectiosum, and hydrops fetalis. Sometimes non­specific URTI may be produced by 
this virus

Educational Objective:
Dry hacking cough (brassy cough, seal’s bark cough), dyspnea and inspiratory stridor 
in a child  with a recent URTI is suggestive of viral laryngotracheobronchitis (croup) 
The most common cause of croup is parainfluenza virus type1. Children with croup 
need to be hospitalized and closely monitored.
74
A 26 ­year­old male from a developing country presents to the clinic with six months 
history of productive cough, night sweats and low­grade fever. His sputum was 
cultured on blood agar and Sabraud’s medium. Creamy white colonies were seen after 
3 days. On microscopy, they show budding yeast forms. When one colony was 
incubated with serum for 2 hours at 37◦ C and examined under the microscope, it 
showed presence of hyphal structures. The most likely site of colonization by this 
organism before it was found in sputum is:

Answers

A. Bronchioles and small bronchi
B. Fibrous cavities in lung
C. Oral cavity
D. Inflamed lung parenchyma
E. Large bronchi and trachea

Explanation:
Presence of budding yeast and formation of hyphal structures on incubation in serum 
(germ tube test) is indicative of fungus of Saccharomycetales group, of which candida 
albicans is most commonly encountered. Candida species is assuming greater 
importance as a human pathogen because of aging population, widespread use of 
immunosuppressive for many diseases and increase in invasive procedures carried out 
on patients. Candidiasis is the fourth commonest cause of nosocomial infection in 
USA. C. albicans accounts for 50% cases of candidiasis, rest is due to non­albicans 
species like C.glabrata, C. parapsilosis and C. krusei. Germ tubes are specific for C 
albicans. Very rarely, they are formed by C. dubliniensis. C. glabrata do not form germ
tubes and C. lusitaniae form very few. 
C. albicans is the most common opportunist mycosis. It is also a frequent colonizer of 
human skin and mucous membranes. (Candida contributes to the normal flora of skin.
mouth. vagina. and intestine.)
Superficial candida infections are associated with antibiotic use, corticosteroid use, 
diabetes mellitus, HIV and other immunosuppressive illnesses. These superficial 
infections include oral thrush, vulvovaginitis and cutaneous candidiasis. Disseminated
candidiasis occurs in neutropenic patients and most often affects the esophagus, heart,
liver and kidney.
Candida does not usually produce lung infestation. This patient's putrefactive 
symptoms as well as the fact that he is originally from a developing country suggests 
possibility of tuberculosis. Presence of candida in his sputum only indicates that his 
oral cavity is colonized. (Choice C)
Choices A and E: The trachea, large and small bronchi are normally sterile

Choice B: Aspergillus fumigatus is a fungus that colonizes already existing lung 
cavities. It forms a 'fungus ball" (aspergilloma) that may prooduce cough and 
hemoptysis.

Choice D: Many pathogens, including bacteria, viruses and fungi, can cause 
inflammation of the lung parenclyna. Candida associated pulmonary disease is 
however uncommon.

Educational Objective:
Candida albicans is a normal inhabitant of the GI tract (including the oral cavity) in 
up to 10% of the population. Thus, it is a common contaminant of sputum cultures. 
Isolation of candida from sputum does not indicate disease.
75
A 6­month­old infant develops meningitis. Gram­negative coccobacilli were seen in his 
CSF on Gram staining. Initially it was not possible to obtain the growth of this 
microorganism on 5%sheep blood agar. However, luxurious growth of Gram­negative 
coccobacilli was seen when CSF was cultured on same media along with a streak of 
Staphylococcus aureus colonies. Staphylococci promote, growth of the bacteria by 
supplementing

Answers:

A. Iron
B. Catalase 
C. NAD.
D. ATP
E. Pyruvate

Explanation:

Subject:   Haemophilus influenzae grown around a streak of Staphylococcus aureus

Subject:  Haemophilus influenzae on chocolate agar plate
Haemophilus influenzae is a blood­loving organism and requires X factor (exogenous 
haemin) and V factor (NAD or NADP) to support its growth. Pure 5% sheep blood agar
lacks sufficient nutrients to support the growth of Haemophilus species. It also does 
not allow growth of haemophilus because of the presence of V factor inactivating 
enzymes (NADase). Growth of Haemophilus influenzae can be achieved on 5% sheep 
blood agar by cross streaking the medium with Staphylococcus aureus. Colonies of H, 
influenzae will grow around the hemolytic S. aureus colonies, a phenomenon known as
satellitism. This occurs because staphylococcus secretes an excess of V factor. Due to 
hemolysis of RBC (produced by staphylococcus), X factor is also released from lysed 
erythrocytes. S aureus therefore provides the X and V factors necessary to support the 
growth of Haemophilus influenzae in sheep blood agar.

Choice B: Catalase is an enzyme present in staphylococcus. Its presence is used to 
differentiate the staphylococci from the streptococci.

Choice D: Pyruvate is the end product of glycolysis and is the starting substrate for 
synthesis of glucose, lactate, fatty acids, amino acids and nucleic acids. It is used by all
pathogenic bacteria in metabolism of sugars for energy.
 
Choice E: Iron is an essential growth factor for many bacteria. It is present in all blood
agar media, as it is derived from iron contained in erythrocytes.

Choice F:  Lactose fermentation as a source of energy is best studied in operation of lac
operon in E. coli. Lactose can bind specifically to the lac repressor, which thus gets 
altered. This alteration allows RNA polymerase to attach to the promoter site and 
transcribe the structural genes of the lac operon.
 
Educational Objective:
Haemophilus influenzae is a "blood loving" organism. Part of the laboratory 
identification process of Haemophilus influenzae is demonstration of its requirement 
for X factor and V factor for its growth. This can be achieved on 5% sheep blood agar 
by streaking the media with staphylococcus aureus. Hemolysis produced by S. aureus 
supplies the X factor, and V factor is secreted in excess by S. aureus itself. The 
phenomenon is known as satellitism  
76
A 5­year­old boy presents to the clinic with history of pain in the ear region. His 
physical examination shows features of acute otitis media. The tympanic membrane is 
ruptured and pus is coming out. Microbiological examination of pus shows growth of 
Haemophilus influenzae. However, patient’s immunization history is up to date and 
includes vaccination for H. influenzae. What explains presence of otitis media caused 
by these bacteria in this patient? 

Answers 
A Cellular immune functions are defective in this patient.
B Patient’s neutrophil functions are defective.
C The strain responsible for this patient's disease produces exotoxin
D  The strain responsible for this patient's disease is unencapsulated
E Effective vaccine against Haemophilus influenzae is not available

Explanation:
Acute otitis media is seen particularly in children between 6 months and 12 years of 
age. In the pediatric age group, S. pneumoniae (35%), H. influenzae (25%) and M. 
catarrhalis (15%) are the predominant bacteria involved. Viruses alone or in 
conjunction with bacteria are seen in 25% of cases. H influenzae is classified, or typed 
into 6 serotypes (a­f) based on the antigenic structure of its polysaccharide capsule. H. 
influenzae that do not possess polysaccharide capsule are called nontypeable. More 
than 90% of H. influenzae strains isolated from middle ear aspirates of infected 
children are non­typeable. Similarly, most cases of chronic bronchitis are associated 
with non­typeable strains. Haemophilus influenzae is found exclusively in humans. It 
resides principally in upper respiratory tract. About 25 to 80% people have non­
capsulate strain and 5 to 10% have capsulate strain in their nasopharynx. Majority of 
capsulate strains belong to serotype b. In unvaccinated individuals, invasive H. 
influenzae infection can culminate in meningitis, epiglottitis, bacteremia, pneumonia, 
septic arthritis and cellulitis. The polysaccharide capsule is the major virulence factor;
hence, immunization with H. influenzae type b conjugate vaccine (Hib vaccine) is 
usually employed. However, Hib vaccine will not prevent infection by non­capsulate 
(non­typeable) strains of H. influenzae. As mentioned earlier these strains are mostly 
implicated in acute otitis media in children (Choice D)

Choice A: Haemophilus influenzae infection despite adequate vaccination does not 
indicate defective cellular immunity as a T­cell response is generally not induced by 
the conjugate vaccine. The conjugate vaccine stimulates B­cells.  
 Choice B: Haemophilus influenzae infection despite immunizations does not signify 
defective neutrophil function, although poor neutrophil function or low neutrophil 
count can predispose to this disease.
Choice C: Exotoxin is not produced by any strain of H. influenzae, typeable or not. The
vaccine for H. influenzae is directed against capsular polysaccharide and not against 
any toxin.

Choice E: The protein­conjugated Hib vaccine is very effective in inciting immunity to 
Haemophilus influenzae type b in children. 

Educational Objective:
Nontypeable strains of Haemophilus influenzae are the strains that do not have 
antiphagocytic polysaccharide capsule. This capsule is the major virulence factor of 
this microorganism; hence, vaccine against the most prevalent capsulated bacteria (i.e.
Haemophilus influenzae type b) is usually employed for vaccination purposes. 
However, immunity against non­typeable strains, as well as for typeable strains other 
than type b, is not conferred by vaccination with the Hib vaccine.
77
A 2­day­newborn develops lethargy, pyogenic skin lesions and respiratory distress. 
Blood culture show growth of beta­hemolytic Gram­positive cocci that are catalase 
negative and bacitracin­resistant. Which of the following measures could have 
prevented this condition?

Answers

A. Penicillin at 30 weeks gestation  
B. Maternal vaccination against group B streptococcus
C. Breast feeding restriction
D. Intrapartum penicillin or ampicillin
E. Postnatal immunoglobulin

Explanation:

The finding of Gram­positive cocci in chains indicates presence of streptococcus, as 
staphylococci classically form clusters. They cab be distinguished further by the 
catalase reaction. Staphylococcus is catalase positive. Group A Streptococci 
(S.pyogenes) and Group B streptococci (S. agalactiae) are beta­hemolytic, but 
bacitracin resistance excludes S. pyogenes (GAS) and indicates S. agalactiae (GBS).
GBS is a leading cause of bacterial sepsis and meningitis in newborn and a major 
cause of endometritis and fever in parturient women. The 2002 guidelines for 
‘Perinatal group B Streptococus Prevention’ recommend universal prenatal screening 
for group B streptococcal colonization of maternal vaginal and rectal tract at 35­37 
weeks gestation. In women positive for GBS or in women who have had an infection by
GBS in the past, intrapartum antibiotic prophylaxis is indicated to prevent neonatal 
GBS sepsis, pneumonia and meningitis. Others recommend antibiotic prophylaxis only
for high­risk patients (preterm delivery, early rupture of membranes, prolonged labor, 
fever or chorioamnionitis). Incidence of group B streptococcal disease in babies less 
than a week old is declining due to these recommendations. Penicillin remains the 
first line agent for intrapartum antibiotic prophylaxis. Ampicillin is an acceptable 
alternative. (Choice D)

Choice A: Early eradication of streptococci (at 30 weeks gestation) will not serve the 
purpose, as the mother would again be vulnerable to reinfection after 4 weeks. 
Choice B: Theoretically, immunization of women against S. agalactiae before or during
pregnancy could prevent intrapartum infection of the baby. It would be preferable 
than antibiotic administration as it would prevent development of resistant strains. 
However, the vaccine is not ready as yet.

Choice C: Breast­feeding should not be restricted under normal circumstances because
the human milk, specially the colostrum, provides some mucosal immune protection to
the newborn. Additionally, breast­feeding is recommended by the American Academy 
of Pediatrics as the sole source of nutrition to all infants for the first six months 
because to its superior nutritional content, its ability to invigorate the proper 
development of the GI tract and for immune protection afforded to the baby by IgA 
antibodies in human breast milk.

Choice E: Postnatal immunoglobulin administration would be a little late, since 
infection can occur at an earlier period of parturition or in late gestation.

Educational Objective:
Universal prenatal screening for group B streptococcus colonization of anogenital 
region by swab culture al 35­37 weeks gestation is recommended to identify infected 
women. They need be given intrapartum antibiotics as prophylaxis to prevent 
neonatal GBS sepsis, pneumonia and meningitis
78
An 8­year­old immigrant boy is brought to the emergency room with a swollen right 
knee accompanied by fever and malaise. His joint movements are limited and very 
painful. His past medical history is not significant Some of his vaccinations are not up 
to date. His synovial fluid and blood culture grew gram­negative rods on hematin 
enriched medium. The microorganism isolated above, is able to produce disease 
because of the presence of which of the following components?

Answers

A. Capsule
B. Endotoxin
C. Fimbriae
D. Hemolysins
E. Hyaluronidase
F. Exotoxin 

Explanation:

H. influenzae is a gram­negative bacillus, which may or may not show presence of a 
polysaccharide capsule. Capsulated strains are subdivided into serotypes designated   
a –f, depending on antigenic structure of their capsule. The six cappsular subtypes can
be identified by polymerase chain rection. Haemophilus influenzae is a blood­loving 
organism and requires both X factor (exogenous hemin) and V factor (nicotinamide 
adenine dinucleotide) for its growth on culture media. Most infections are caused by H.
influenzae serotype b. Type b is the only H. influenzae  whose capsule contains ribose 
rather than hexose sugar and this feature may be related to the virulence of the 
organism. The type b capsular material is a polymer consisting of ribose, ribitol and 
phosphate, and is known as polyribosyl­ribitol phosphate (PRP). The PRP capsule 
prevents phagocytosis and intracellular killing by neutrophils and is essential for 
virulence of this organism (Choice A). Antibodies to serotype b are shown to promote 
opsonization, complement fixation and phagocytosis of H. influenzae. 
Unencapsulated (nontypeable) H. influenzae are part of the normal flora of the upper 
respiratory tract. They can produce only local infection such as sinusitis, otitis media 
or bronchitis. Local disease is more likely in presence of an underlying abnormality. 
Without the benefit of a capsule, these strains are unable to produce invasive diseases.

Choice B:  Being gram­negative organisms, H. influenzae possess LPS endotoxin in 
their cell membranes, but this endotoxin is not a major virulence factor. Endotoxin is 
the major virulence factor in infections caused by N. meningitidis, Salmonella and E. 
coli.

Choice C: Fimbriae are not present on H. influenzae. These proteinaceous projections 
from bacterial cells mediate attachment to target tissues during the process of 
establishing infection. Fimbriae play an important role in N. gonorrheae, N 
meningitidis and E.coli infection.

Choice D: Hemolysins are not produced by Haemophilus influenzae

Choice E: The enzyme hyaluronidase is not present in Haemophilus influenzae. This 
enzyme is important for pathogenesis of infections caused by Staphylococci and 
Streptococci (group A and C).

Choice F: There are no strains of H. influenzae known to produce an exotoxin. 

Educational Objective:
The pathogenicity of H. influenzae is dependent on the presence of the antiphagocytic 
polysaccharide capsule. Type b is the most invasive and virulent strain of 
Haemophilus influenzae. It has a capsule with ribose as the sugar rather than a 
hexose. Unencapsulated (nontypeable) strains of these bacteria are part of the normal 
flora and cause only local infections.
79

A 3­year­old child has history of recurrent staphylococcal skin infections. Now he is 
presenting with a liver abscess due to aspergillus infection. The patient is thought to 
have a mutation of a structural component of a neutrophilic enzyme system that is 
involved in microbicidal activities. This child would have increased vulnerability to 
infections caused by

Answers 

A. Giardia lamblia
B. Herpes simplex type I
C. Mycobactrium tuberculosis
D. Crypcococcus neoformans
E. Streptococcus  pyogenes 
F. Corynebacterium diphtheriae
G. Pseudomonas cepacia

Explanation:

This patient has chronic granulomatous disease (CGD), a condition caused by 
mutation in genes encoding neutrophil NADPH oxidase system. NADPH oxidase is 
involved in production of H2O2 , which itself is capable of destroying bacteria, albeit 
slowly or it may enter H2O2 ­ MP­halide system to form potent microbicidal substance, 
hypochlorite. Patients with defects in NADPH oxidase are unable to generate 
sufficient H2O2 , specially when the infecting organism is capable of producing 
catalase. Patients with CGD are at increased risk for infections caused by: 
Staphylococcus aureus, Pseudomonas cepacia (Burkholderia cepacia), Serratia 
marcescens, Nocardia species, Aspergillus species and Chromobacterium violaceum. 
These organisms are all catalase positive and catalase decomposes H 2O2 (2H2O2 → O2 + 
2H2O)
In CGD patients, the production rate of H2O2 and the downstream microbicidal HOCL 
is imperiled.  It appears that catalese positivity in infecting organismsis necessary but 
not sufficient for opportunistic infections to occur, since the risk of infection with other
catalase positive microbes (such as M. tuberculosis, C.neoformans, and C. diptheriae) 
is not increased in CGD (Choice G).

Choice C, D and F: All these microorganisms are catalase positive, but patients of 
CGD do not have increased susceptibility to these organisms. This indicates that other
factors apart from catalase positivity are also essential for infectivity in a patient of 
CGD.
Choice A, B and E: These microorganisms are catalase negative. Presence of catalase 
enzyme is one of the essential prerequisites for microorganism to be able to produce 
disease in patients of CGD.

Educational Objective:
Chronic granulomatous disease (CGD) results from a genetic defect in NADPH oxidase
enzyme system. Normally NADPH oxidase participates in the killing of microbes 
within neutrophil phagolysosomes. Patents with CGD develop recurrent pulmonary, 
cutaneous, lymphatic and hepatic infections with a tendency for granuloma formation, 
usually beginning in childhood. The infections are predominantly caused by S. aureus, 
P. cepacia (Burkholderia cepacia), Serratia marcescens, Nocardia species, Aspergillus 
species and Chromobacterium violaceum.
80
A 40­year­old Asian immigrant presents to your clinic with complaints of productive 
cough, low­grade fever, night sweats and occasional hemoptysis. Sputum cultures 
placed on a selective medium grow mycobacterium. On microscopic examination, the 
bacteria are growing as parallel chains. This observed growth pattern, often correlates
with which particular property of the bacteria?

Answers

 Acid­fastness         
 Virulence
 Multi­drug resistance
 Ability to survive within macrophages.
 Growth rates

 
Explanation:
The clinical picture of this patient is typical of pulmonary tuberculosis. The growth 
pattern of bacteria in vitro, as alluded to in the question, is described as serpentine 
cords. It is indicative of elaboration of cord factor by these microorganisms. Cord factor
is a glycolipid (trehalose 6, 6’­dimycolate). Experimental injection of this glycolipid in 
mice is seen to inhibit neutrophil migration. It also promotes granuloma formation. In 
vivo, cord factor is associated with the virulence of these bacteria. Mycobacterium that
do not possess this factor are unable to cause disease. More specifically, cord factor is 
responsible for inactivating neutrophils, damaging mitochondria and inducing release 
of tumor necrosis factor. Another virulence factor believed to be elaborated by 
mycobacteria is sulphatides. Sulphatides inhibit fusion of secondary lysosomes with 
mycobacteria­ contain phagosomes (Choice B).

Choice A:  Acid­fastness is a properly inherent to all mycolic acid­containing 
microorganisms including Mycobacterium and Nocardia.

Choice C: multiple drug resistance in mycobacteria arises due to spontaneous point 
mutation in the genome. It is usually the result of monotherapy or due to poor 
compliance with the treatment.

Choice D: Ability to survive within macrophages is a function of sulphatides as 
described earlier. 
Choice E: Growth rate is not responsible for the formation of cords by mycobacteria. 

Educational Objective:
Virulent mycobacteria will grow as "serpentine" cords in vitro. This property is 
dependant on elaboration of trehalose 6, 6’­dimycolate by the bacteria and it correlates
with virulence of these microorganisms.
81

If a sample of contaminated soil is heated to 100°C for 15 minutes, which of the 
following bacteria is likely to survive this treatment.

Answers

A. Bacillus anthracis
B. Streptococcus pyogenes
C. Escherichia coli
D. Pseudomonas aeruginosa
E. Listeria monocytogenes

Explanation:

Usually members of Bacillus and Clostridium family are the spore­forming bacteria 
that are present in contaminated soil. Spores are resistant to heat and can survive 
temperatures of 100°C. Hence, members of Bacillus and Clostridium family that are 
present in soil would be able to withstand heating of soil up to 100°C. Spores not only 
withstand high temperatures but they can also resist desiccation and disinfecting 
chemical agents. Thus, autoclaving is the only method to destroy them (Choice A).

Choice B, C, D and E: These organisms do not form spores and would be killed if 
heated upto 100°C

Educational Objective:
Spore­forming bacteria survive boiling. Bacillus anthracis and some members 
Clostridium family are potentially pathogenic bacteria. They are found in soil and are 
capable of forming spores.
82
A 45­year­old male presents with features of advanced HIV infection (marked weight 
loss, CD4 cell count < 100 cells/cmm). He complains of productive cough and chest pain
for which a bronchoscopy is done. His broncholalveolar lavage (BAL) fluid shows 
presence of budding yeast forms with thick walls. These yeast cells stained intensely 
with mucicarmine stain. Infection with this organism may also more commonly 
manifest as

Answers:

A. Interstitial pneumonia 
B. Oral plaques
C. Esophagitis
D. Meningitis
E. Skin infection
F. Sinusitis

Explanation:
Yeast forms whose capsule stain with mucicarmine stain is most often Cryptococcus 
neoformans. Trichosporon beigelii and Blastomyces dermatidis are the other fungi 
that show staining with mucicarmine. B. dermatidis shows broad based budding and 
retracted cytoplasm. Trichosporon usually does not infect the lungs and has a different
morphology. 
C. neoformans affects only immunocompromised patients. Generally, it is found in soil 
contaminated by pigeon droppings. Usually it is acquired by inhalation; person­to­
person transmission does not occur. Risk factors other than HIV infection are diabetes,
immunosuppressive therapy, neoplasia and immunologic diseases. Following 
inhalation the disease may remain localized in the lung or it may disseminate to 
distant sites, depending on immune status of the patient. In immunodeficient 
individuals, lungs shows diffuse involvement and is often accompanied by other 
infections like P.carinii or CMV. The disease may spread to CNS where it produces 
cryptococcal meningitis, which usually appears insidiously. Bone and joint infection 
lead to abscess formation. Skin involvement presents as papules that ulcerate later on.
 Presumptive diagnosis can be obtained by staining the CSF with India ink and it 
shows a distinct halo (due to presence of capsule) around yeast form. More specific 
latex agglutination tests and EIA assays are also available (Choice D)

Choice A: Cryptococcal pulmonary disease produces granulomas. It usually does not 
cause interstitial pneumonia. 
Choice B: White oral plaques are a common manifestation of Candida infection (oral 
thrush). 

Choice C: Esophagitis in an immunodeficient patient is most commonly caused by 
Candida infection

Choice E: Cryptococcus may cause cutaneous mycosis that manifests with papules, 
Pustules, nodules and ulcer. However, cutaneous cryptococcal disease is rare and 
occurs in about 10% of cases.

Choice F:  Mucormycosis classically affects the paranasal sinuses. Mucormycosis 
occurs in immunocompromised patients and is strongly associated with diabetes 
mellitus and diabetic ketoacidosis. Aspergillus is another fungus that may cause 
sinusitis.

Educational Objectives:
Meningitis is the most common presentation of cryptococcus neoformans infection, 
which occurs in immunosuppressed patients. It can be diagnosed by India ink staining
of the CSF. Cryptococcal pneumonia is diagnosed by mucicarmine staining of lung 
tissue and BAL fluid.
83
A 44­year­old male is having HIV infection from last eight years. He is on 
antiretroviral therapy for last 6 years. Now, he presents to the hospital with 
complaints of cough and low­grade fever. He has lost 10 pounds of weight in last 6 
months. On examination, he has hepatospenomegaly. A light microscopic image of his 
bone marrow aspirate is shown below.

What organism is seen in the graphic above?

Answers

A. Sporothrix schenkii
B. Coccidiodes immitis
C. Candida albicans
D. Aspergillus fumigatus
E. Cryptococcus neoformans
F. Histoplasma capsulatum
Explanation:
This image shows small ovoid bodies within a macrophage. Of the choices listed, only 
Histoplasma capsulatum can survive intracellularly and produce systemic disease. In 
immunocompetent individuals, H. capsulatum infection is either asymptomatic or it 
may produce a self­limiting pulmonary disease in form of a granulomatous 
inflammation. In immunodeficient individuals and rarely in young patients, this 
organism may produce an acute disseminated disease. This can be rapidly progressive 
and fatal. Rarely a chronic form of disseminated disease may be seen in previously 
healthy persons.  Disseminated histoplasmosis causes hepatosplenomegly because of 
its predilection for the reticuloendothelial system. Ulcerated lesions on the tongue are 
also very characteristic of disseminated histoplasmosis.
Chest x­ray of a patient with disseminated histoplasmosis may show diffuse 
pulmonary infiltrates with marked lymphadenopathy. In chronic lung disease, the 
radiographic changes resemble those of pulmonary tuberculosis. Examination of lung 
biopsy specimens and bone marrow aspirates reveals oval or round yeasts like forms 
within macrophages. They need to be distinguished from cells of C. glabrata, B.  
dermatitidis and P. marneffei. As H. capsulatum is a dimorphic fungus, culture on 
Sabrauds medium will show growth of hyphal form. Histoplasma antigen can be 
assayed in blood or urine and is useful both for diagnosis and for monitoring treatment
response. (Choice F)

Choice A: Sporothrix schenkii is a dimorphic fungus. Yeast form shows a characteristic
cigar shaped bud. It usually produces lesions on the skin. In immunodeficient persons,
disseminated disease may be seen. 

Choice B: Coccidioides immitis also causes disseminated mycosis in immunodeficient 
patients. In tissue sections, it is seen as large spherules containing endospores.

Choice C: Candida albicans is the most frequent opportunistic fungal agent. It is 
extracellular and forms yeast and pseudohyphae.

Choice D: Aspergillus fumigatus is a monomorphic fungus. It is seen in tissues as 
septate hyphae with V­shaped branching.

Choice E: Cryptococcus neoformans shows yeast cells that stain intensely with 
mucicarmine stain.

Educational Objective:
H. capsulatum can survive intracellularly within macrophages. It causes a 
disseminated mycosis in immunocompromised patients and constitutes one of the 
AIDS defining conditions. The clinical features include systemic symptoms (fever and 
weight loss), painful oral ulcers, lymphadenopathy and hepatosplenomegaly,

84
Some animals were given subcutaneous injection of certain Gram­positive bacteria. An
antibiotic was then administered to them. After a few days, certain gram­positive 
bacilli were isolated from the wound site. When placed in a hypotonic solution, these 
microorganisms underwent rapid disintegration due to swelling. Which of the 
following antibiotic was used in this experiment?       

A. Ciprofloxacin
B. Cefuroxime 
C. Chloramphnenicol
D. Azithromycin
E. Doxycycline

Explanation:

Gram­positive organisms have a cytoplasmic membrane composed of a phospholipid 
bilayer as well as a peptidoglycan cell wall outside of cell membrane. The 
peptidoglycan wall provides the shape to the bacterium as well as resistance to 
osmotic stress. They also protect the bacterium agents the immune system and in 
many instances play an important role in pathogenecity of these organisms.  Under 
normal circumstances and within a certain range, Gram­positive organisms would not 
be destroyed by changes in tonicity of the surrounding environment, because of their 
intact peptidoglycan wall. In the experiment described above, the bacteria were 
destroyed when placed in a hypotonic solution. Therefore, it can be safely inferred that
these bacteria had some deficiency in their peptidoglycan cell wall. The only antibiotic 
mentioned in the list above that acts against cell wall synthesis is Cefuroxime. This is 
a second­generation cephalosporin. Cephalosporin’s are beta­lactam antibiotics and 
are related to penicillin. Vancomycin is another example of an antibiotic that inhibits 
cell wall synthesis (Choice B)

Choice A: Ciprofloxacin belongs to fluoroquinolone class of antibiotics. These 
antibiotics act by inhibiting bacterial DNA gyrase and topoisomerase IV, thereby 
leading to bacterial DNA damage

Choice C: Chloramphenicol is an anti­ribosomal antibiotic that acts to inhibit bacterial
ribosomes. Its use is limited by its toxicity, which includes a dose­related 
myelosuppression leading to anemia as well as aplastic anemia, the most feared side­ 
effect of this drug

Choice D: Azithromycin is a macrolide antibiotic that acts by inhibiting the bacterial 
50 S ribosomal subunit. This inhibits protein chain formation in the bacteria

Choice E:  Doxycycline is an antibiotic that acts by inhibiting bacterial 30S ribosomal 
subunit and therefore bacterial protein synthesis.

Educational Objective:
Penicillins, Cephalosporins and Vancomycin are able to disrupt the peptidoglycan cell 
wall of Gram­positive and Gram­negative organisms. Exposure of bacteria to these 
antibiotics depletes their cell wall. As a consequence, such bacteria are unable to 
handle osmotic stresses. 
85
A 30­year­old man develops an ulcerative lesion on the skin over his shinbone from 
last two months. Base of the ulcer is covered with dirty white exudate.Microscopic 
examination of exudate shows acute inflammatory exudate, necrotic material and few 
ill­defined granulomas. Histologicalexamination after silver methenamine stain shows
presence of yeast cells with a single broad based bud. Another rpiece of tissue from the
same specimen was placed in a specific medium at 24○ C. this procedure showed 
growth of hyhal forms. The patient is most likely infected with

Answers
A. Blastomyces dermatitidis
B. Malessezia furfur
C. Cryptococcus  neoformans 
D. Aspergillus fumigatus
E. Rhizopus species 
F. Candida  glabrata 
Explanation:
Certain fungal species display variable morphology when growing in different 
environment at different temperatures and alternate between the yeast and the mold 
forms. Such fungi are called dimorphic. They grow as hyphal structures at 25­30°C 
and as yeasts at body temperature (36 ­37°C). In tissues, they are usually seen as 
yeasts. Four species of dimorphic fungi are of medical significance. Their 
characteristics are shown in the following table. 
Of the possibilities listed above, only B.dermatitidis is a dimorphic fungus (Choice A)

Choice B: Malassezia furfur produces cutaneous mycosis. Skin scrapings show short 
hyphae and spores, an appearance that is likened to ‘spaghetti and meatballs’

Choice C: Cryptococcus neoformans shows yeast forms in clinical specimens. Yeasts 
show intense staining with mucicarmine stain. Pseuedohyphae may be seen.

Choice D:  Aspergillus fumigatus shows only septate hyphal form in tissues along with
‘fruiting forms’. It mainly affects immunocompromised patients.

Choice E: Rhizopus, Mucor and Absidia are the saprophytic fungi that cause 
mucormycosis in immunosuppressed patients. It shows broad non­septate hyphae in 
tissue specimens

Choice F: Candida glabrata is a component of normal human flora. It causes 
disseminated infection in immunocompromised individuals. In tissue specimens it is 
seen as budding yeast with pseudohyphae

Educational Objective:

Dimorphic fungi grow as molds at 25­30°C and as yeast at body temperature (35­37C). 
Medically important dimorphic fungi include Sporothrix, Coccidioides, Histoptasma 
and Blastomyces species.
86
A 40­ year­old Caucasian male presents with a history of long standing cough and 
weight loss. His chest x­ray shows presence of pulmonary infiltrates. His past medical 
and personal history is unremarkable. A transbronchial biopsy was obtained and it 
shows presence of granulomas. Which of the following agents can produce this clinical 
picture?

Answers

A. Mucor species
B. Candida albicans 
C. Pneumocystis jiroveci (carinii)
D. Blastomyces dermatitidis
E. Aspergillus fumigatus 

Explanation:
Of the choices listed above, only Blastomyces dermatitidis can produce granulomatous 
inflammation in an immune competent individual. Endemic areas of this fungus are 
areas adjacent to Mississippi and Ohio River, areas northwest of the great lakes in 
Canada and areas within Africa.  A history of travel to an endemic area can often be 
elicited. There are no specific risk factors for infection with B. dermatitidis, although 
HIV infection predisposes to disseminated form of this disease. The fungus exists in 
environment in mold form, found in soils’ organic matter. The infection is transmitted 
by inhalation of the fungus from soil. Person­to­person transmission is not seen. After 
entering the lungs, it transforms into yeast form (thermal dimorphism).

Blastomyces dermatitidis may be asymptomatic in an immunocompetent person or it 
may produce a flu­like illness or a more severe infection with productive cough and 
hemoptysis. The infection may become chronic, when it is characterized by granuloma 
formation. Infection can spread by hematogenous route to skin, subcutaneous tissue, 
bones and oropharynx. Sputum examination shows round yeast with thick, refractile 
walls. Yeast may show a single, broad­based bud. In­situ hybridization tests for tissue 
sections and complement fixation tests for serum are also available (choice D).

Choice A: Mucormycosis (Zygomycosis) usually produces rhinocerebral disease. 
Sometimes it may manifest as a pulmonary disorder and presents as pneumonia. On 
histology, the fungus is angioinvasive and elicits acute necrotizing inflammatory 
response. Granulomas are absent. 
Choice B: Candida albicans is an opportunistic pathogen. It may cause a superficial 
infection in an immunocompetent host (e.g. vulvovaginitis). Systemic disease, 
however, occurs only in the immunosuppressed patients.

Choice C: Pneumocystis jiroveci causes pulmonary disease in immunocompromised 
patients like HIV infected individuals or transplant recipients on immunosuppressive 
drugs. 

Choice E: Aspergillus fumigatus is an opportunistic pathogen. It causes invasive 
aspergillosis in immunocompromised patients, in patients with old lung cavities and in
asthmatic patients.

Educational Objective:

Blastomyces dermatitidis can cause pulmonary disease in the immunocompetent 
hosts. In immunocompromised individuals, it may lead to disseminated mycosis.
87
A 40 ­year­ old Caucasian male presents with chronic symptoms referable to his 
respiratory system. Microscopic examination of lung tissue shows mixed inflammatory
cell infiltrate alongside spherules packed with endospores. Which of the following 
events is likely to be present in this patient’s history?

Answers

A. Chemotherapy for leukemia
B. Cave exploration in Ohio 
C. Travel   to Arizona
D. Exposure to pigeon droppings
E. Previous history of cavitary tuberculosis 
F. Long history of bronchial asthma

Explanation:
Chronic lung disease due to an infection that shows presence in tissue of spherules 
packed with endospores is indicative of Coccidiodes immitis infection. This is a 
dimorphic fungus that has a mold form (hyphae) in cultures at 25 ○ C and a yeast form 
(spherules with endospores} at body temperature (37­40○ C). it I a fungus endemic in 
USA (Lower Sonoran Life zone of California, Arizona, Nevada, Utah and New Mexico),
Mexico and South American peninsula.
C. immitis is transmitted by inhalation of spores (arthoconidia). Once inside the lungs,
the spores turn into spherules that contain endospores. The spherules subsequently 
rupture and release endospores that disseminate to other organs and tissues. The 
disease may develop in up to 40% individuals who inhale these spores from 
contaminated soil. C. immitis causes lung disease that may be asymptomatic or it may
produce flu­like symptoms. Skin lesions can also occur (erythema nodosum or 
erythema multiforme). Immunosuppressed patients may develop systemic 
coccidiodomycosis (Choice C)

Choice A: Neutropenic patients are at high nsk for developing opportunistic mycosis by
Candida albicans, Aspergillus fumigatus and Mucor. 

Choice B:  Histoplasma capsulatum is endemic to the Mississipi and Ohio River 
basins. It is found in bird and bat droppings. Patients with histoplasmosis often have a
history of caving.

Choice D: The yeast form of cryptococcus neoformans is present in pigeon droppings. 
This fungus causes pulmonary disease and meningoencephalitis in immunodeficient 
patients.
Choice E: Aspergillus fumigatus can colonize old lung cavities (e.g. formed by 
tuberculosis) to form a ‘fungal ball’ (aspergilloma). 

Choice F: Patients of bronchial asthma are at risk of developing an allergic reaction to 
Α fumigatus called allergic bronchopulmonary aspergillosis. 

Educational Objective:
Coccidioides immitis is a dimorphic fungus endemic to the southwestern U.S. It exists 
in the environment as a mold that forms spores. These spores are inhaled and turn 
into spherules in the lungs.
88
A group of biologists had gone on exploration of some caves near Ohio River in eastern 
United States in search of a new species of birds. After about 6 months, some of them 
present to the hospital with fever, cough, chest pain and malaise. Pulmonary 
infiltrates and hilar lymphadenopathy was seen on chest x­rays. A transbronchial lung
biopsy was done and a fungus was detected. What morphological form is likely to be 
seen in this fungal disease?

Answers

A. Budding yeast with a thick capsule
B. Septate hyphae with dichotomous branching
C. Ovoid cells within macrophages
D. Pseudohyphae and blastoconidia
E. Multinucleated spherules

Explanation:
History of cave exploration in eastern USA preceding onset of a pulmonary disease 
should raise suspicion of infection with the fungus, Histoplasma capsulatum.             
H. capsulatum is a thermally dimorphic fungus and is amongst the commonest agent 
to produce a fungal disease in endemic areas. Other thermally dimorphic fungi are 
Blastomyces dermatitidis, Coccidiodes immitis and Sporothrix schenkii. Each of these 
fungi has a restricted geographical distribution in USA. Knowledge about geographical
localization of different fungi can help in arriving at the correct diagnosis.
H. capsulatum is found as a mold in soil. It is also present in bird and bat droppings
It is transmitted by the respiratory route when bird or bat droppings containing fungal
spores are inhaled. In lungs, the fungus is ingested by macrophages, and is seen on 
light microscopy as small intracellular oval bodies (Choice C). Cellular immune 
reactions are provoked by presence of this fungus and the usual outcome is a 
granulomatous tissue response. Because the fungus targets histiocytes and the 
reticuloendothelial system, it may cause lymphadenopathy and hepatosplenomegaly.
Extent of exposure to the causative agent and immune status of the patient will 
determine the clinical manifestation to this fungus. In immunodeficient persons, 
disseminated histoplasmosis can occur. In otherwise healthy individuals, 
asymptomatic infection, acute flu­like syndrome or protracted lung disease that 
eventually heals by calcification can be seen. Some other manifestations of infection 
are granulomatous and fibrosing mediastinitis and chronic pulmonary histoplasmosis.
Choice A: Cryptococcus neoformans  takes the form of budding yeast with a thick capsule. 
This yeast also grows abundantly in soil containing bird (pigeon) droppings 
However; this fungus tends to cause disease  in immunocompromised individuals.

Choice B: Aspergillus fumigatus causes pulmonary disease in immunocompromised 
persons. This fungus is seen in tissue sections as septate hyphae with V­shaped 
branching

Choice D: Candida species are yeasts that form pseudohyphae. Blastoconidia are spores
that grow as buds on fungal hyphae. Candida infection is usually not associated with 
pulmonary infiltrates or lymphadenopathy

Choice E:  Spherule packed with endospores is found in coccidioides immitis infection, 
Like Histoplasma, coccidioides can cause pulmonary disease in immunocompetent 
persons. Coccidioides is endemic to the southwestern U.S.

Educational Objective:

Histoplasma capsulatum is a thermally dimorphic fungus that in tissue sections is 
seen as intracellular yeasts within macrophages. It affects the lungs and 
reticuloendothelial system. It is present in bird and bat droppings, and is endemic to
the Mississippi and Ohio River basins.
89
A 35­year­old male with past history of pulmonary tuberculosis that was treated 
adequately, now presents to your office because of occasional hemoptysis. Apical region
of his left lung had to be resected and it showed presence yellowish gray friable 
material filling a fibrous cavity, but the mass was not attached to the wall of the 
cavity. On microscopy, friable mass consisted of a tangled collectin of hyphae. This 
patient’s condition would best be described as:

Answers

A. Colonizing
B. Disseminated
C. Contagious
D. Allergic
E. Necrotizing 
F. Invasive

Explanation:

A ball of fungal elements in a cavitary lesion of lung is most likely to be due to 
colonization by Aspergillus fumigatus. Rarely fungal balls can be produced by 
Pseudallescheria, Sporothrix and Coccidioides. Aspergillus fumigutus is a mold that is 
widely present in organic matter. It forms septate hyphae that branch at 45° angles 
(V­shaped branching). Normally, when spores of Aspergillus are inhaled, they are 
cleared by mucus and ciliated epithelium of the respiratory tract. However, in 
presence of cavitary lesions in the lung, in an immunocompetent individual (E.g. old 
tubercular cavities, bronchiectatic cavities, bronchial cyst etc), inhaled spores of          
Α. fumigatus may colonize the dead space and grow to form a fungal ball 
(aspergilloma). On chest x­ray, an aspergilloma will appear as a radiopaque structure 
that shifts when the patient changes his position. In aspergilloma, tissue invasion is 
not seen (Choice A). In immunocompromised individuals, Aspergillus fumigatus may 
produce disseminated disease with involvement of lung, brain and other organs. In 
such persons, the fungus can also gain entry through abraded skin and then gets 
disseminated to other organs. Hypersensitivity reactions to aspergillus occur in 
patients with asthma, producing a condition called allergic bronchopulmonary 
aspergillosis. Invasive aspergillosis occurs in immunosuppressed and neutropenic 
patients.
Choice B: In immunodeficient persons, Aspergillus may invade blood vessel wall and 
then spread hematogeneously.

Choice C: Aspergillosis is not contagious. Aspergillus fumigatus is present in the 
environment and causes opportunistic infection in immunocompromised individuals.

Choice D: Allergic bronchopulmonary aspergillosis occurs in patents with asthma.

Choice E: In immunocompromised individuals, invasive aspergillosis can produce 
necrotizing lesions of surrounding tissue

Choice F Invasive Aspergillus can become disseminated Aspergillus hematogenously 
and can affect any organ system.

Educational Objective:

Aspergillus fumigutus causes opportunistic infections in immunodeficient and 
neutropenic patients (invasive aspergillosis). Invasive aspergillosis can become widely 
disseminated. Aspergillus fumigatus can also colonize old cavities producing 
aspergilloma. In asthmatic patients, it produces allergic bronchopulmonary 
aspergillosis.
90
A 44­year­old, HIV positive man presents with complaints of sudden­onset fever, 
chills, cough, and left­sided chest pain that worsens on deep breathing. Physical 
examination reveals features of pulmonary consolidation in lower part of the left lung. 
No other infection is present. His most recent CD4 T lymphocyte count is 760 
cells/cmm. He is on antiretroviral therapy.  Which of the following organisms may be 
responsible for his respiratory symptoms?

Answers

A. Mycobacterium tuberculosis 
B. Mycoplasma  pneumoniae 
C. Streptococcus pnuemoniae 
D. Pneumocystis  jiroveci
E. Legionella  pneumophila
F. Staphylococcus aureus
G. Moraxella  catarrhalis 

Explanation:

This patient is having symptoms referable to the respiratory system, which can be 
explained by presence of lobar consolidation seen on his chest x­ray. His HIV positive 
status should make one vigilant to be on lookout for the presence of opportunistic 
infections. It is unlikely that the patient is having pulmonary tuberculosis because of 
acute onset of his respiratory symptoms. His pneumonia can be because of 
encapsulated bacteria to which HIV positive patients are especially susceptible. Or it 
may be due to opportunistic pathogens like P. carinii, fungi or some viruses. Patients’ 
CD4 cell count is within normal range (normal = 400 to 1400 cells/cmm). Opportunistic
pulmonary infections in HIV positive individuals tend to occur when CD4 cell count 
falls below 200 cells/cmm. This suggests that his pneumonic state is more likely to be 
caused by encapsulated bacteria. HIV positive patients are especially prone to develop 
infections with S. pneumoniae and H. influenzae. This is because of deranged B cell 
function and functional defects in neutrophils, secondary to altered cellular immunity. 
It is observed that HIV positive patients have six­fold increase in the incidence of 
pneumococcal pneumonia, and this may be seen in patients with relatively intact 
immune system (Choice C).
Choice A: M. tuberculosis is not likely in this patient as his symptoms are acute in 
nature.

Choice B, D and E: These organisms are opportunistic pathogens and usually tend to 
produce infection when CD4 cell count is below 200 cells/cmm.

Choice F: M.catarrhalis infection tends to occur in patients older than 50 years, those 
with long history of cigarette smoking and those with underlying diseases like COPD.

Educational Objective:
The most common cause of community acquired pneumonia in immunocompetent 
individuals (and that includes HIV positive patients with normal CD4 cell count) is 
streptococcus pneumonia. HIV positive patients are susceptible to pulmonary infection
by capsulated microorganisms.
91
A 1­year old infant presents to the hospital with symptoms pertaining to oral thrush, 
pneumonia, wasting and severe lymphopenia. His mother has intravenous drug 
addiction problems. She did not avail any prenatal care during her pregnancy. Nor has
the child undergone any postnatal evaluation. Now her mother wants‘quick fix’ for her
child’s problems. She refuses to have any investigations performed on herself or her 
child. Which of the following prenatal interventions is likely to have prevented this 
malady?

Answers:

A. Inteferons 
B. Prenatal vitamins
C. Live attenuated vaccine
D. Viral component vaccine 
E. Viral enzyme inhibitor drugs 
F. Cellular receptor inhibitor drugs
G. Killed vaccine

Explanation:

The infant’s present predicaments of oral thrush, wasting and pneumonia, in a setting 
of severe lymphopenia and a drug addict mother, is highly suggestive of HIV infection 
and its vertical transmission to the baby. The risk of HIV infection occurring in an 
infant, born to an HIV­positive mother, who received no prenatal and postnatal 
antiretroviral therapy, is estimated to be between 15 to 30%. Higher rates of vertical 
transmission of HIV are seen in presence of high maternal viral load, low CD4+ 
counts, low HIV p24 antibody levels, presence of STDs  and other factors that expose 
the fetus to mother’s blood and genital secretions for prolonged period. If maternal 
HIV load is greater than 100,000 copies of HIV RNA per milliliters of blood, the rate of
vertical transmission is 40%. Risk of vertical transmission can be reduced by 
zidovudine treatment of HIV infected mother, beginning in the second trimester of 
pregnancy and continued up to 6 weeks following birth. Other measures recommended
are, minimal exposure of fetus to mother’s secretion and blood during childbirth, 
avoidance of breast feeding and intermittent administration of ART to the infant 
(Choice E).

Choice A: Interferon therapy is useful in treatment or hepatitis B and C virus 
infection, hairy cell leukemia, condyloma acuminatum and Kaposi sarcoma. Its role in 
prevention of HIV infection is not known.

Choice B: Maternal vitamin A deficiency is associated with higher rates of vertical 
transmission of HIV. But vitamin A supplements during pregnancy will not completely
eradicate chances of vertical transmission.

Choices C, D and G: Currently no effective vaccination is available against HIV 
infection.

Choice F: HIV virus gains entry to the host cells (mostly CD4+ T­lymphocytes) with 
the help of gp121 and gp41 molecule on its surface that binds to the CD4 molecule on 
the host cell. However, studies proving efficacy of fusion inhibitor drugs in prevention 
of vertical transmission of HIV, are lacking.

Educational Objective:
Studies have shown that ART prophylaxis during pregnancy with the nucleoside 
analogue zidovudine, a retroviral reverse transcriptase inhibitor, reduces the risk of 
vertical transmission of HIV infection.
92

A 30­year­old man with a past history of laparotomy, following motor vehicle accident 
4 years ago, develops fulminant infection. His blood culture shows growth of 
Streptococcus pneumoniae. He dies despite best efforts to eradicate the infection. 
Which protective mechanism was most likely to be defective in this patient that 
contributed to the dismal outcome?  

Answers

A. Phagocyte function 
B. Complement fixation
C. Interferon release
D. Opsonization
E. Bacterial clearance         
F. Intracellular killing of bacteria

Explanation:

The patient had a motor vehicle accident for which an emergency laparotomy was 
performed. It is likely that a spleenctomy was done, which will explain this patient’s 
susceptibility to pneumococcal infection. Apart from many other functions that the 
spleen sub serves, one of its important roles is to provide assistance to the body in 
eliminating microorganisms. 
The white pulp of the spleen consists of lymphoid tissue, phagocytes and dendritic 
cells. These cells contribute in ingesting antibody­ coated bacteria and presenting 
them to T­cells residing in spleen, in order to quickly mount an immune response. 
Nearly half of the body's total immunoglobulins are produced by the splenic B­
lymphocytes. A significant portion of neutrophils of the sequesterd pool is normally 
present in spleen. They are promptly available when need arises.
Asplenic patients are predisposed to sepsis with encapsulated bacteria such as S. 
pneumoniae, H. influenzae, and N. meningitidis. This happens because of decreased 
ability to recognize and clear these organisms (Choice E). Vaccination against these 
three organisms is recommended for asplenic patients. Apart from these pyogenic 
bacteria, asplenic patients are also susceptible to infection by unusual organisms like 
Babesia and Capnocytophaga canimorsus. Increased susceptibility to infection in 
asplenic patients may be seen even 25 years after the loss of this organ.
Choice A: Defective phagocytic function is seen in Chediak­ Higashi syndrome. In this,
there is reduced fusion of phagosomes to lysosomes, an essential step in microbial 
degradation. 

Choice B: Deficiencies of key components of the complement cascade can lead to 
increased susceptibility to infection. Deficiency in formation of membrane attack 
complex is for unknown reason, associated with a high incidence of infection by only 
N.meningitidis.  
Choice C: Interferon is released from virus­infected cells. It produces activation of 
macrophage and cytotoxic T­Lymphocytes. Defects in interferon release lead to 
increased susceptibility to viral infections.

Choice D: Opsonization is the process by which antibodies and complement make 
foreign material more prone to phagocytosis and destruction. Spleen is not involved 
per se in the opsonization process.

Choice F: Defect in intra cellular killing of microorganisms is typically seen in Chronic
Granulomatous Disease (CGD). Such patients are more prone to infection by catalase 
positive organisms

Educational Objective:

The spleen serves both as a site of antibody synthesis and as a reservoir of phagocytic 
cells capable removing circulating pathogens. Asplenic patients are more prone to 
infection caused by encapsulated organisms such as S. pneumoniae, H. influenzae and
N. meningitides.
93
A 25­year­old female who has been a recipient of a kidney transplant two years back 
develops fever, cough and chest pain. X­ray chest shows pulmonary infiltrates in lower
part of the left lung. She is put on broad­spectrum antibiotics. As there is no 
significant improvement in her condition, a bronchoscopy is done. Bronchoalveolar 
lavage fluid obtained during the procedure is examined microscopically. Some yeast­
like structures were seen, which stained intensely with mucicarmine stain, as shown 
below. What is the likely diagnosis?

Answers:

A. Coccidiodes immitis
B. Candida albicans 
C. Rhizopus species
D. Aspergillus fumigatus 
E. Cyptococcus neoformans
F. Blastomyces dermatitidis
G. Histoplasma capsulatum
Explanation:  

Fungi showing positive staining by mucicarmine stain are Cryptococcus neoformans, 
Trichosporon beigelii and Blastomyces dermatitidis. Mucicarmine stains the 
polysaccharide capsule of C. neoformans. The capsule is a major virulence factor of 
this pathogen. In addition to positive mucicarmine staining, C. neoformans shows 
narrow based budding. 
C. neoformans usually affects immunocompromised patients, usually those with 
defects in cell­mediated immunity. It is transmitted via the respiratory route and may 
cause pulmonary disease. Involvement of the lung is usually asymptomatic, but 
pneumonia­like symptoms may be produced. Chest x­ray findings are non­specific, and
may show infiltrates or nodules. Diagnosis is made by identifying the fungus in 
sputum, bronchoalveolar washings, or tissue samples. Gomori’s methenamine silver 
(GMS) and mucicarmine stains are used for identification. However, unencapsulated 
variants will not stain with mucicarmine. They are stained by Masson’s Fontana stain.
C. neoformans is a neurotropic fungus. Subacute or chronic meningo­encephalitis is 
the most common neurological manifestation of this disease. Prognosis is largely 
dependant on immune status of the patient. In severely immunocompromised persons,
mortality may approach 50%. It is very difficult to eradicate this organism in AIDS 
patient (Choice E)

Choice A: Coccidiodes immitis causes lung disease in immunocompetent people and 
disseminated mycosis in immunocompromised individuals. In tissue samples, it 
appears as large, irregularly sized, thick­waved spherules that contain multiple 
endospores.

Choice B: Candida albicans shows yeast like forms and pseudohyphae.

Choice C: Rhizopus shows broad non­septate hyphae with branching at right angles.

Choice D: Aspergillus fumigatus branching septate hyphae and fruiting buds.

Choice F: Blastomyces dermatitidis cause both lung disease and disseminated mycosis.
Microscopically, it appears as round yeast with broad­based budding and a thick, 
doubly­refractile wall.

Choice G: Histoplasma capsulatum is a dimorphic fungus that causes tuberculosis­like
pulmonary disease. It is faintly stained in tissue sections appearing as multiple small, 
ovoid yeast forms in histiocytes.
Educational Objective:

Cryptococcus neoformans is a pathogenic fungus that has a polysaccharide capsule. 
The capsule appears red on mucicarmine stain and as a clear unstained zone in India 
ink preparation. Few other fungi also stain by mucicarmine stain, but their 
morphology is different

94
A 38­year­old male suffering from bronchial asthma shows presence of proximal 
bronchiectasis and pulmonary infiltrates on his chest x­ray. His differential blood 
count shows 18% eosinophilia. He has recurrent episodes of dyspnea, which is relieved 
by inhalation of bronchodilators. His serum shows presence of precipitating antibodies
to a certain microorganism. This patient's condition is most likely due to colonization 
by:

A. Legionella pneumophila
B. Strongyloides stercoralis
C. Pesudomonas aeruginosa
D. Streptococcus pneumoniae
E. Adenovirus
F. Aspergillus  fumigatus

Explanation:

Presence of eosinophilia, proximal bronchiectasis and pulmonary infiltrates indicate 
presence of a subacute or chronic inflammatory pathology due to a hypersensitivity 
reaction. Of the choices listed above, Aspergillus fumigatus is the organism most likely
to colonize cavities and produce ongoing inflammation, thereby exacerbating tendency 
for proximal bronchiectasis. Aspergillus fumigatus is a low virulence fungus that 
generally does not cause significant infections except in immunocompromised or 
debilitated patients. It may however colonize the bronchial mucosa and complicate 
asthma or cystic fibrosis via a hypersensitivity reaction. The result is allergic 
bronchopulmonary aspergillosis (ABPA). ABPA occurs in 5% to 10% of steroid­
dependent asthmatics. Patients with this condition have very high levels of serum IgE,
eosinophilia and IgE plus IgG serum antibodies to Aspergillus. There is intense airway
inflammation and mucus plugging with episodes of exacerbations and remissions. 
Repeated exacerbations may produce transient pulmonary infiltrates and proximal 
bronchiectasis (Choice F).
Choice A: Legionella commonly produces lesions in immunocompromised hosts. 
Bronchial asthma is not a predisposing factor for this infection.

Choice B: Strongyloidiasis can cause pulmonary symptoms including transient 
pneumonia as the larvae are migrating through lungs during completion of their life 
cycle. They may produce eosinophilia, but bronchiectasis is not a complication of this 
parasitic infestation.

Choice C: Pseudomonas aeruginosa is an aerobic Gram­negative bacillus that is a 
frequent and sometimes a deadly, pulmonary pathogen in patients with cystic fibrosis 
or neutropenia. In patients of cystic fibrosis, it may be isolated from lungs in almost all
cases.

Choice D: Uncomplicated pneumococcal lung disease usually resolves completely 
without causing structural changes in lung. Hence, presence of bronchiectasis and 
eosinophilia is unlikely to be caused by S. pneumoniae.

Choice E: Adenovirus can exacerbate asthmatic symptoms, but per se, they do not 
produce bronchiectasis. 

Educational Objective:
Allergic bronchopulmonary aspergillosis (ABPA) due Aspergillus fumigatus may 
complicate bronchial asthma. The disease may produce transient but recurrent 
pulmonary infiltrates that eventually end up as proximal bronchiectasis.
95

There has been an outbreak of methicillin resistant Staphylococcal infection in a 
hospital, which is suspected to be of noscomial origin. Laboratory personnel are 
instructed to collect sample from hospital staff for purpose of identification of carriers. 
Sample from which site would be most appropriate for this purpose?

Answers:

A. Oropharynx
B. Hands
C. Anterior nares
D. Perineurn
E. Axilla

Explanation:
Humans are a natural reservoir for S aureus. More than 90% of staphylococci are 
resistant to penicillin and more than 50% have become methicillin resistant (MRSA).  
Resistance to methicillin is mediated by mecA gene, which encodes for an altered 
penicillin binding protein. This allows bacterial wall peptidoglycan synthesis in 
presence of methicillin. MRSA can be hospital acquired or community acquired. 
Colonization of the nasopharynx is most common site in asymptomatic carriers. It is 
this site that is sampled most often by epidemiologists, wishing to study S. aureus 
carrier state. In the general population, 25­ 30% of individuals have nasal colonization 
with staphylococcus. Nasal carriage increases the risk noscomial infections in 
hospitals, particularly in intensive care or surgical units.

Educational Objective:
In the general population at any given time, 25­30% of individuals have nasal 
colonization with MRSA. The anterior nares are the most common site of colonization 
for both methicillin­sensitive and methicillin resistant S. aureus
96

A 20­year­old pregnant woman, who is a migrant from a developing country, develops 
low­grade fever and maculopapular rashes during the first trimester. The rash is seen 
first over her face, and it then spreads to the trunks and extremities in the next 48 
hours. Physical examination shows post­ auricular lymphadenopathy. There is mild 
thrombocytopenia. The rashes lasted for about 3 days. Due to this disease, some 
immediate complication can be seen in the mother (first column) and certain 
congenital anomaly may occur in the infant (second column). Identify the correct row.

Answers
        
                Mother                                   Infant

A. Polyarthralgia                                      macrocephaly
B. Meningitis                                            malformed teeth
C. Polyathralgia                                  deafness
D. Deafness                                               bow legs
E. Pneumonia                                           cataracts

 
Explanation:
 
Rubella and Rubeola (measles) are two acute exanthematous viral diseases whose 
rashes begin on the face and spread downwards. In view of the patient’s immigration 
history, the possibility of her not being vaccinated against mumps, measles and 
rubella need to be kept in mind. Clinical findings of a patient with rubella include 
maculopapular rashes, posterior cervical and post­auricular lymphadenopathy, fever 
and coryza. Polyarthritis occurs in about 25% of the patients. Rubella without rashes 
is also possible. Rubella, as such does not produce significant morbidity. Post 
infectious encephalopathy may occur rarely. However, occurrence of Rubella in 
pregnancy particularly in first trimester can have devastating effect on the fetus. 
Fetal infection in first trimester leads to congenital rubella in 80% of cases. Congenital
rubella can produce early onset cataracts, micropthalmia, and glaucoma, hearing 
defects, congenital heart diseases and psychomotor retardation. The diagnosis can be 
confirmed by a specific test for IgM rubella antibody or by isolation of the virus
(Choice C)

Choice A, B, D and E: Rubella infection does not produce meningitis, deafness or 
pneumonia in adults. Congenital rubella does not show macrocephaly, bowlegs or 
malformed teeth.

Educational Objective:
 
Maternal rubella infection produces a low­grade fever, a maculopapular rash with 
cephalocaudal progression, and posterior, post­auricular and suboccipital 
lymphadenopathy.  25% of adult women patients develop polyarthritis. Polyarthralgia 
can be seen in greater proportion. Congenial rubella syndrome is associated with 
sensory­ neural deafness, cataracts, and congenital heart diseases.
97
A 31­year old female presents to the fertility clinic with complaints of inability to 
conceive. She is married for last three years and is not using, contraceptives of any 
kind. Her male partner has been evaluated and no abnormality is detected. Prior to 
her marriage, the woman was using oral contraceptives for past 12 years. She had 
lower urinary tract infection 3 years ago for which ceftriaxone was given for 5 days.  
No other significant history is elicited. What can be the possible reason for her 
inability to conceive?

Answers 

A. Advanced matemal age
B. Conception attempted during inappropriate time of menstrual cycle.
C. Inadequaute antibiotic therapy
D. Oral contraceptive­induced ovarian failure
E. Anti­sperm antibodies

Explanation:
Tubal disease is responsible for about 20% cases of infertility due to causes pertaining 
to the female partner. The most common cause of infertility related to tubal causes is 
pelvic inflammatory disease (PID). PID is most frequently caused by N. gonorrheae 
and Chlamydia trachomatis infection. Other microorganisms include anaerobes, 
Haemophilus influenzae, enteric gram­negative bacilli and streptococci. In many parts
of the world, M. tuberculosis is an important pathogen in causation of PID. Infection 
by N.gonorrhoeae/ C. trachomatis can often be asymptomatic. When symptomatic, 
they will initially cause a purulent urethritis followed by ascension of the infection to 
the cervix. Infection that is more widespread can involve endometrium, fallopian tubes
and peritoneal cavity. Thin cervical mucus, vaginal douching, and open cervical os 
during menstruation tend to encourage ascension of infection. PID can also occur 
secondarily to peritoneal infection like acute appendicitis or due to hematogenous 
spread as in tuberculosis. Invasive intrauterine procedures, intrauterine devices or 
hysterosalpingography also contribute in causation of PID.
Treatment of gonococcal PID must always include treatment for C. trachomatis as 
well.  A third generation cephalosporin will treat the gonococcal infection and further 
treatment by clindamycin or doxycyclin for C. trachomatis is indicated. Metranidazole 
may be given for anaerobic infection. In this case, the patient was only treated with a 
cephalosporin to which C.trachomatis is not responsive. This can lead to tubal 
inflammation, scarring and infertility (Choice C). 
 
Choice A:  Advanced maternal age becomes a factor in the ability to conceive after the 
age of 35 years. The decrease in fecundity after this age in many women is due to 
aging of the oocytes.
 
Choice B: A woman is most likely to conceive when cohabitation is around the 
ovulation time. As the women is trying to conceive for last three years, this factor 
must have been taken into account

Choice D: Oral contraceptive use is not associated with ovarian failure. Ovarian 
failure can occur approximately 2 years earlier in women who are smokers. Treatment
like radiation therapy and chemotherapy can induce ovarian failure 
 
 Choice E: Anti­sperm antibodies may be present in male serum or seminal fluid or 
female serum and cervical mucus. They may occur in testicular trauma, viral orchitis 
or after bacterial infection of genitourinary tract. However, the history in this couple 
does not support this etiology.
 
 
Educational Objective:
Pelvic inflammatory disease is most frequently caused by N gonorrheae and 
C. trachomatis. If urethritis due to these pathogens is not treated adequately, it may 
lead to PID with subsequent scarring of fallopian tube and infertility. 
98
A 30­year­old immigrant from Africa presents to the clinic after getting his fingers 
burnt, though he does not feel much pain at the scalded site. On examination, 
proximal to the burnt site, some hypopigmented patches are present. The ulnar nerve 
is found to be thickened. A nerve biopsy was done and it showed granulomatous 
inflammation with presence of many bacteria invading schwann cells. What is the 
likely causative organism?
 
Answers  

A. Campylobacter fetus
B. Borellia burgdorferi 
C. Corynebacterium diphtheriae
D. Mycobacterium leprae
E. Treponema pallidum

Explanation:
Leprosy or Hansen’s disease is a deforming infection primarily of the skin and nerves 
that is caused by Mycobacterium leprae. Transmission is believed to occur through the
respiratory route but direct cutaneous contact has not been excluded as a mode a 
transmission. Infection is also seen on coming in contact with armadillo in southwest 
USA.  Leprosy has a wide range of clinical manifestations that vary depending on the 
extent of cell­mediated immune response to the organism.
Most severe form of leprosy is called lepromatous leprosy. It occurs in patients with a 
weak cell­mediated immune response to the bacteria. As a result, leprae bacilli may 
disseminate widely in the body. M. leprae grows best at temperatures that are slightly
lower than core body temperature. This explains primary localization of the disease to 
the skin, superficial nerves, eyes and testis. Lepromatous leprosy manifests clinically 
as diffuse skin thickening and cutaneous hypopigmentation and plaques (often 
accompanied by loss of hair), leonine facies, paresis and regional anesthesia of motor 
and sensory nerves, testicular destruction and blindness.
The least severe form of leprosy is often self­limited and is called tuberculoid leprosy. 
In this subtype, the infection is limited by an intact cell­mediated immune system. 
Mild skin plaques develop that are associated with hypopigmentation, loss of hair 
follicles, and focally decreased sensation. Features intermediate between the 
tuberculoid and lepromatous forms are often seen (Choice D). 

Choice A: Campylobacter fetus is a Gram­negative rod responsible for mild enteritis in
immunocompetent patients and mild systemic bacteremic illness in 
immunocompromised patients
Choice B: Borrelia burgdorferi is the spirochete responsible for Lyme disease. 
Symptoms of Lyme disease are a characteristic skin rash, fever, myalgias and malaise.
Systemic disease can progress to cause arthritis, facial paresis and cardiac 
manifestations
 
Choice C: Corynebacterium diptheriae is a gram­ negative rod responsible for 
diphtheria.

Choice E:  Trepopema pallidum is the spirochete responsible for syphilis. Skin 
involvement is in form of chancres, gummas or plaques depending on the stage of the 
disease. It does not produce skin anesthesia.
  
 
Educational Objective:
Leprosy or Hansen disease is a systemic illness caused by Mycobacterium leprae. The 
severity of disease depends on the extent of the cell­mediated immune response to the 
bacteria. .
99
A 30­year­old male presents to the clinic complaining of fever, malaise, inguinal 
lymphadenopathy, pain in joints and maculopapular rashes over the body. The rashes 
are more prominent on hands and feet. During investigation, a test was carried out in 
which, patients serum was added to a suspension containing cardiolipin, lecithin and 
cholesterol. Within minutes, extensive flocculation was seen. What should be the 
approach in management of this patient?
 
Answers:
A. Investigate for the presence of bacterial toxins
B. Investigate for the presence of cold agglutinins
C. Investigate for the presence of rheumatoid factor 
D. Investigate for the presence of spirochaetal antibodies 
E. Investigate for the presence of fungal antibodies 

 
Explanation:
The investigation described above shows presence of nontreponemal antibodies in 
serum of the patient. Commonly it is known as VDRL test or rapid plasma reagin test.
It is positive in late primary, secondary and tertiary syphilis but may be negative in 
early primary, late latent and late tertiary syphilis. Apart from syphilis, the test may 
also be positive in acute viral illnesses, in infection by M.pneumoniae, Chlamydia, and
malaria and sometimes in late pregnancy. If this test remains positive for longer than 
6 months, apart from untreated syphilis, conditions that need to be considered are, 
collagen vascular diseases, hepatitis­C infection and leprosy.
This test is considered as a nontreponemal serologic test because it does not detect 
treponemal organisms or antibodies directed against treponemal organisms. Instead, 
it detects antibodies to human cellular lipids released into the bloodstream due to cell 
destruction caused by T pallidum. Because these tests are easy to perform and are 
inexpensive, they are the first line of investigation in suspected cases of treponemal 
infection. If a nontreponemal test is positive, confirmation is obtained by carrying out 
specific treponemal test like fluorescent treponemal antibody absorption test (FTA­
ABS) (Choice D).
 

Choice A: T. pallidum do not produce toxins, hence investigating for toxins would not 
be of much help.

Choice B: Cold agglutinins are characteristic of M. pneumoniae infection. Presence of 
cold agglutinins can be tested by placing patient’s plasma at 4○ C in a refrigerator. It 
will show clotting. 

Choice C: Rheumatoid factor (RF) is an anti­IgG antibody used in evaluation of 
patients with autoimmune diseases, especially rheumatoid arthritis (RA). 

Choice E: Fungal antibodies can be used as a method of diagnosing some acute 
infection caused by fungi (e.g. Histoplasmosis)

Educational Objective:
In patients suspected to be suffering from treponemal disease, non­specific 
nontreponemal antibody tests like VDRL or RPR are first employed as a screening 
investigation. Confirmation is obtained by utilizing more specific tests like, FTA­ABS 
and TPHA.  
 

100
A 4­year­old Asian immigrant is brought to clinic with complaints of high fever, nasal 
discharge, dry cough, excessive tear formation and discomfort in bright light. When 
examined, his oral mucosa shows presence of white spots against an erythematosus 
background. What other physical findings are likely to appear in this patient within 2­
3 days?

Answers
 
A. Jaundice
B. Bronchiolitis
C. Profuse diarrhea 
D. Parotitis
E. Skin rash

Explanation:
This patient’s clinical picture raises suspicion of infection by rubeola virus (measles). 
Rubeola virus is a member of paramyxovirus family. Some salient features of this 
infection are:
1. Nearly all non­immune infected individuals develop the disease.
2. Following an episode of measles, persons show nearly complete resistance to 
re­infection.
3. It is a highly contagious disease and nearly all susceptible children are likely 
to contract the disease on exposure.
4. Only one antigenic type of virus is seen.
5. In malnourished children having poor medical facilities, the disease can be life
threatening.

Some common clinical manifestation of the disease are, cough coryza, conjunctivitis 
and Koplik’s spot. Koplik’s spots appear about 2 days before appearance of rashes. 
Rashes are maculopapular; appear first on face followed by trunk and extremities. In 
malnourished children, measles can be complicated by bacterial superinfections and 
pneumonia. Rarely encephalitis and subacute sclerosing panencephalitis may develop
(Choice E)
 

Choice A: Hepatitis viruses can cause jaundice but not typically associated with cough,
rhinitis or buccal lesions,

Choice B:  Occasionally, patients with rubeola with develop pneumonia or 
laryngotracheobronchitis (croup). Bronchiolitis is typically not seen. Respiratory 
syncytial virus (RSV) can cause an upper respiratory tract infection that progresses to 
bronchiolitis, but RSV is not associated with buccal lesions.
 
 
Choice C: Rotavirus often causes an upper respiratory tract infection followed by 
diarrhea.

Choice D: Measles is not commonly associated with parotitis. Parotitis is seen in 
mumps.
 
 
Educational Objective:
 
The tetrad of cough, coryza conjunctivitis and Koplik’s spots (CCCK) is diagnostic of 
measles (rubeola) infection. Koplik’s spots are tiny white or blue­gray lesions on the 
buccal mucosa that precede the maculopapular skin rash by 2­3 days.

 
 

También podría gustarte